VIQ - TEST PAPER 4 Flashcards

1
Q

@# 1.A 71-year old man with a long history of alcohol-related liver cirrhosis showed a subtlebut equivocal area of abnormality on a screening ultrasound. His AFP was raised at 2,000.An MRI of the liver with contrast is performed. All of the following are expected featuresof infiltrative HCC on MRI, except

A. Hyperintense on MR images acquired during the hepatobiliary phase after injection ofhepatocyte-specific contrast agent.

B. A reticular appearance of the tumour can be seen during the venous and equilibriumphase.

C. Washout appearance of the tumour is usually reported as irregular andheterogeneous.

D. Infiltrative HCC may commonly appear as iso- or hypointense on images obtainedduring the arterial phase.

E. Infiltrative HCC may be difficult to discern from underlying heterogeneous cirrhosisbecause of its permeative appearance.

A

1.A. Hyperintense on MR images acquired during the hepatobiliary phase after injection of
hepatocytes specific contrast agent

At contrast-enhanced CT and MR imaging, infiltrative HCC may be difficult to discern from underlying heterogeneous cirrhosis because of its permeative appearance, its minimal and inconsistent arterial enhancement and the heterogeneous washout appearance that occurs during the venous phase.

The enhancement pattern of infiltrative HCC seen on images obtained during the hepatic arterial phase has been reported as minimal, patchy or miliary.

Although arterial hyperenhancement is a key diagnostic feature of nodular and massive HCC, infiltrative HCC may commonly appear as iso- or hypointense on images obtained during the arterial phase.

Washout appearance is a specific CT and MR imaging feature of typical nodular HCC.

Hypointensity relative to the surrounding liver parenchyma during the venous phase of enhancement remains a valid sign for the detection of infiltrative HCC.

However, washout appearance of the tumour is usually reported as irregular and heterogeneous and is less frequently seen in infiltrative HCC than in other HCC subtypes.

Moreover, a reticular appearance of the tumour has been seen on images obtained during the venous and equilibrium phases, possibly related to fibrosis.

Finally, the tumour generally appears as hypointense on MR images acquired during the hepatobiliary phase after injection of hepatocyte specific contrast agent because of the lack of contrast agent uptake.

How well did you know this?
1
Not at all
2
3
4
5
Perfectly
2
Q

@# 7. A 77-year-old man with weight loss and deranged LFTs had an ultrasound scan that showed multiple liver lesions suspicious for metastases. Contrast-enhanced CT of the chest and abdomen was done in search of the primary’. Which one of the following is the most common primary tumour that has hypovascular liver metastases?
A. Pancreas
B. Stomach
C. Colon
D. Kidney
E. Melanoma

A
  1. C. Colon
    Liver metastases may be hypovascular or hypervascular.

Colon, lung, breast and gastric carcinomas are the most common tumours causing hypovascular liver metastases, and they typically show perilesional enhancement.

Neuroendocrine tumours (including carcinoid and islet cell tumours) renal cell carcinoma, breast, melanoma and thyroid carcinoma are the tumours most commonly causing hypervascular hepatic metastases, which may develop early enhancement with variable degrees of washout and peripheral rim enhancement.

Statistically, colonic metastases are more common than gastric carcinoma metastasis to the liver.

How well did you know this?
1
Not at all
2
3
4
5
Perfectly
3
Q

@# 8. A 65-year-old woman is recovering from a double lung transplant On the fourth day post-transplant, she starts complaining of shortness of breath. Clinical findings include some basal crackles; air entry seems satisfactory in the upper zones. An urgent chest X-ray is organised. Portable up-to-date chest X-ray shows evidence of pulmonary’ oedema. What is the most likely cause for this appearance?
A. Barotrauma
B. Acute graft failure
C. Volume overload
D. Response to antirejection treatment
E. Reimplantation response

A
  1. E. Reimplantation response
    Ischaemia reperfusion injury or reimplantation response is a non-cardiogenic pulmonary oedema that typically occurs more than 24 hours after transplantation, peaks in severity on post-operative Day 4, and generally improves by the end of the first week. The oedema may continue up to 6 months post-operative; however, in most lung transplant recipients, it has cleared completely by 2 months. The radiographic and HRCT features are non-specific and may include perihilar ground-glass opacities, peribronchial and perivascular thickening, and reticular interstitial or airspace opacities located predominantly in the middle and lower lung lobes.
    Acute rejection due to a cell-mediated immune response commonly occurs in the second week post-operative. HRCT features are also relatively non-specific and may include ground-glass opacities (often with basal distribution), peribronchial cuffing, septal thickening and pleural effusion.
How well did you know this?
1
Not at all
2
3
4
5
Perfectly
4
Q

@# 9. Cryptorchidism has an increased risk of development of all of the following testicular tumours, except
A. Leydig cell tumour
B. Seminoma
C. Yolk sac tumour
D. Embryonal cell tumour
E. Choriocarcinoma

A
  1. A. Leydig cell tumour

Cryptorchidism is associated with an increased risk of testicular germ cell tumours, more so with bilateral undescended testes. Germ cell tumours are seminomatous or non-seminomatous, which are embryonal cell tumour, yolk sac tumour, choriocarcinoma and teratoma.

Sertoli cell and Leydig cell tumours are non-germ cell tumours.

How well did you know this?
1
Not at all
2
3
4
5
Perfectly
5
Q

@# 10. A 15-year-old teenager presents with a worsening pain in his right ankle. He is an active sports player and plays football for the school club. A radiograph reveals an undisplaced fragment in the medial aspect of the talar dome with a small lucent line around it.
All of the following are well-recognised locations for osteochondral lesions or defects, except
A. Lateral aspect of the medial femoral condyle
B. Medial aspect of the talar dome
C. Lateral aspect of the medial tibial plateau
D. Anterior aspect of the capitellum
E. Humeral head

A
  1. C. Lateral aspect of the medial tibial plateau
    Osteochondral defect (OCD) (previously called osteochondritis dissecans) may result directly from trauma or secondarily from loss of blood supply to an area of subchondral bone, resulting in avascular necrosis. The overlying cartilage, which is nourished by synovial fluid, remains intact to variable degrees. As the necrotic bone is resorbed, the overlying cartilage loses its support. Without its cartilage cover, the bony fragment may become dislodged into the joint.

Common sites of involvement include the lateral aspect of the medial femoral condyle, followed by the talar dome (posteromedial more than anteromedial), anterolateral aspect of the capitellum and the tibial plafond. Rarer sites include navicular, femoral head, humeral head, glenoid and scaphoid.

MRI Grading of OCD
I. Marrow oedema (stable).
II. Articular cartilage breached. Low-signal rim surrounding fragment indicates fibrous attachment (stable).
III. Pockets of fluid (high signal on T2-weighted images) around undetached and undisplaced osteochondral fragment (unstable).
IV. Displaced osteochondral fragment (unstable).

How well did you know this?
1
Not at all
2
3
4
5
Perfectly
6
Q

@# 11. A 35-year-old woman with pre-eclampsia underwent a CT of the brain to exclude intracranial haemorrhage. The CT revealed low attenuation in the white matter of the posterior aspect of both cerebral hemispheres. The abnormal area appeared low on T1W and high on T2W images and was isointense on DWI. No contrast enhancement was evident. What is the diagnosis?
A. Periventricular leukomalacia
B. Progressive multifocal leukoencephalopathy
C. Encephalitis
D. Reversible posterior leukoencephalopathy syndrome
E. CNS lymphoma

A
  1. D. Reversible posterior leukoencephalopathy syndrome
    Reversible posterior leukoencephalopathy syndrome also known as posterior reversible encephalopathy syndrome (PRES), is most commonly encountered in association with acute hypertension, preeclampsia or eclampsia, renal disease, sepsis and exposure to immunosuppressants. At CT or MR imaging, the brain typically demonstrates focal regions of symmetric hemispheric oedema. The parietal and occipital lobes are most commonly affected, followed by the frontal lobes, the inferior temporal-occipital junction and the cerebellum. Lesion confluence may develop as the extent of oedema increases. MR DWI was instrumental in establishing and consistently demonstrating that the areas of abnormality represent vasogenic oedema. The oedema usually completely reverses. Focal or patchy areas of PRES vasogenic oedema may also be seen in the basal ganglia, brainstem and deep white matter (external/internal capsule). When they accompany hemispheric or cerebellar PRES, it is easy to recognise these. Present in isolation or when the hemispheric pattern is incompletely expressed (partial/ asymmetric), diagnosis of PRES can be challenging. If cerebellar or brainstem involvement are extensive, hydrocephalus and brainstem compression may occur. Focal areas of restricted diffusion (likely representing infarction or tissue injury with cytotoxic oedema) are uncommon (11%-26%) and may be associated with an adverse outcome. Haemorrhage (focal haematoma, isolated sulcal/subarachnoid blood or protein) is seen in approximately 15% of patients.
How well did you know this?
1
Not at all
2
3
4
5
Perfectly
7
Q

@# 17. A 35-year old woman with bilateral facial nerve palsy showed extensive nodular deposits with diffuse enhancement of the meninges on CECT of the brain. Which of the following imaging investigation will likely confirm the diagnosis?
A. MR angiogram of the circle of Willis
B. Ultrasound of the liver
C. Intravenous urogram
D. Plain chest radiograph
E. Plain radiograph of both hands

A
  1. D. Plain chest radiograph
    Central nervous system involvement is seen in 5% of patients with systemic sarcoidosis (neurosarcoidosis). The most common parenchymal abnormality described in some series is multiple non enhancing periventricular white matter lesions, seen as high signal intensity on T2-weighted images.
    Enhancing parenchymal mass lesions are also commonly reported. These lesions maybe mistaken for primary or metastatic tumour or tumefactive demyelination. Enhancing mass lesions are frequently associated with nearby leptomeningeal involvement. Leptomeningeal involvement is perhaps the most typical manifestation of central nervous system sarcoidosis, seen in about 40% of cases. This is usually seen as thickening and enhancement of the leptomeninges on contrast- enhanced T1 -weighted images. The enhancement may be diffuse or nodular. Leptomeningeal involvement around the hypothalamus and pituitary infundibulum may be seen with basilar leptomeningeal involvement or as an isolated finding. Cranial nerve involvement is also described. Any cranial nerve can be affected, but the most common cranial nerve deficit involves the facial nerve (VII), whereas radiographically the optic nerves (II) arc most commonly abnormal.
How well did you know this?
1
Not at all
2
3
4
5
Perfectly
8
Q

@# 18. A 10-month-old infant attends the local infectious diseases unit with his mother who recently emigrated from Zimbabwe. The child has shortness of breath, fever and bilateral inspiratory crackles. Chest X ray demonstrates diffuse bilateral ground-glass opacification. What is the most likely diagnosis?
A. Varicella pneumonia
B. Round pneumonia
C. Pneumocystis pneumonia (PCP)
D. Bronchopulmonary dysplasia
E. Congestive cardiac failure

A
  1. C. Pneumocystis pneumonia (PCP)
    PCP is the most common opportunistic infection in immunosuppressed children, occurring in up to 90% of HIV-positive patients. A clinico-pathological and radiological continuum has been reported since the earliest documented cases of PCP. At one end of the spectrum are children with a florid clinical course, who progress from health to death in days. They show marked hypoxia and rapid radiographic evolution of parahilar granular infiltrates to extensive bilateral airspace opacification. Pathologically, an extensive foamy alveolar exudate is seen. Conversely, there are those with an insidious presentation, less profound hypoxia and a slower recovery. These patients tend not to progress to alveolar opacification but show persistent bilateral granular or ground-glass opacification, representing relative prominence of interstitial pulmonary involvement.
How well did you know this?
1
Not at all
2
3
4
5
Perfectly
9
Q

@# 22. In the case of a vertebral compression fracture, all the statements regarding imaging findings suggests a malignant cause, except
A. Involvement of the posterior elements
B. Persistent loss of T1W bone marrow signal on sequential imaging
C. Paravertebral soft-tissue component
D. Post-contrast gadolinium enhancement
K. Convex posterior border of the vertebral body

A
  1. D. Post contrast gadolinium enhancement
    A convex posterior border of the vertebral body is more frequent in metastatic compression fractures than acute osteoporotic compression fractures. A higher frequency of abnormal signal intensity of the pedicle of metastatic fractures is seen in comparison to acute osteoporotic fractures; posterior element involvement is observed more commonly in metastatic compression fractures in comparison to benign fractures. Although epidural mass was suggestive of metastatic fractures, a paraspinal mass is more commonly associated with metastatic compression fracture. Metastatic involvement of other vertebra is also more likely to suggest malignant compression fracture.
    Spared normal bone marrow signal intensity of the vertebral body is highly suggestive of acute osteoporotic compression fractures. Band like low signal intensity on T1-weighted and T2-weighted images is more common in acute osteoporotic compression fractures than metastatic compression fractures. Retropulsion of a posterior bone fragment is more frequent in osteoporotic compression fractures than metastatic compression fractures, although multiple fractures are more commonly benign. Post-contrast enhancement is seen in both malignant and benign causes.
How well did you know this?
1
Not at all
2
3
4
5
Perfectly
10
Q

@# 23. All of the following are true of toxoplasmosis of AIDS, except
A. It is the most common focal CNS infection.
B. Treatment is started empirically based on imaging.
C. The basal ganglia and cerebral hemispheres are commonly involved.
D. Haemorrhage and calcification are common post-therapy.
F. . A single lesion is the most common.

A
  1. E. A single lesion is the most common.
    Toxoplasmosis is the most common opportunistic CNS infection in patients with AIDS. Toxoplasmosis typically manifests on CT scans and MRIs as nodular (small-encephalitis) and/or ring-enhancing (large-abscess) lesions within the brain parenchyma. The enhancing ring, when present, may be somewhat thicker and more ill defined than that seen in association with a typical bacterial abscess.
    The lesions are associated with surrounding oedema and tend to be multiple at presentation. However, a significant percentage of patients present with solitary lesions. Toxoplasmic lesions are most often seen in the basal ganglia and grey-white interface of the cerebral hemispheres. On non- enhanced Tl weighted MR images, the lesions are of low signal intensity; on T2-weighted MR images, the lesions are mildly to moderately hyperintense in relation to the brain parenchyma and can be difficult to separate from the surrounding oedema. Therapy is often begun empirically as soon as CT scans or MR images show focal parenchymal lesions of any sort because the infection is so common in this population. The presence of small haemorrhages may be a sign of toxoplasmosis, and calcifications can occasionally be seen in treated lesions.
How well did you know this?
1
Not at all
2
3
4
5
Perfectly
11
Q

@# 24. On newborn heel stick screening, a newborn infant is found to suffer from congenital hypothyroidism. On ultrasound, the thyroid gland is diffusely enlarged, and on Technetium 99m thyroid scintigraphy there is increased uptake of radioactive tracer within the gland. There is no evidence of ectopic thyroid tissue.
What is the most likely cause for the congenital hypothyroidism?
A. Thyroid hypoplasia
B. Hypothalamic dysfunction
C. Thyroid dyshormonogenesis
D. Maternal antibody induced hypothyroidism
E. Hypopituitarism

A
  1. C. Thyroid dyshormonogenesis
    Thyroid dysgenesis is the most common cause for congenital hypothyroidism, accounting for up to 85% of cases (causes include ectopy, aplasia and hypoplasia); however in this case, the ultrasound and scintigraphy findings do not suggest this as a cause.
    The second most common reason is therefore thyroid dyshormonogenesis, accounting for 10%-15% of cases (also described as thyroid hormone biosynthetic defect, e.g., hereditary Pendred’s syndrome). The remainder of the causes listed are extremely rare.
How well did you know this?
1
Not at all
2
3
4
5
Perfectly
12
Q

@# 25. An 8-year-old child with right upper-abdominal pain and a palpable mass was sent for US of the liver. The US showed a large heterogeneous mass occupying at least half of the right lobe of the liver. The rest of the liver was normal in appearance. Which one of the following is the most common benign primary liver tumour in children?
A. Hepatocellular carcinoma
B. Hepatoblastoma
C. Angiosarcoma
D. Infantile haemangioendothelioma
E. Cholangiocarcinoma

A
  1. D. Infantile haemangioendothelioma
    Although primary hepatic neoplasms represent only a small percentage of solid tumours that occur in children, the finding of focal hepatic lesions in a child is not an uncommon event in a busy radiology practice. The most common neoplasm involving the liver in children, as in adults, is metastatic disease. Most primary liver tumours in children are malignant, but one-third are benign; benign lesions may be of mesenchymal or epithelial origin. The most common benign tumours are, in decreasing order of frequency, infantile haemangioendothelioma, PNH, mesenchymal hamartoma, nodular regenerative hyperplasia (NRH) and hepatocellular adenoma.
How well did you know this?
1
Not at all
2
3
4
5
Perfectly
13
Q

@# 26. A 43-year-old woman with history of proteinuria presented to the A&E department with shortness of breath and haemoptysis. Chest X-ray showed diffuse airspace opacities and an HRCT was organised. The HRCT showed diffuse airspace opacities, ground-glass change and confluent consolidation, which was reported as acute diffuse alveolar haemorrhage. Which of the following can be classically expected on CT sinus study?
A. Bilateral frontal mucocoele
B. Unilateral sinusitis involving all the sinuses on one side
C. Chronic destructive sinusitis with nasal perforation
D. Hypoplastic maxillary sinuses
E. Oro-antral fistula

A
  1. C. Chronic destructive sinusitis with nasal perforation
    Wegener’s granulomatosis is a probable autoimmune disease characterised by systemic necrotising granulomatous destructive angitis, with a classic triad of respiratory tract inflammation, systemic small vessel vasculitis and necrotising glomerulonephritis.
    Sinus disease classically includes destructive sinusitis, nasal septal ulceration, septal perforation and saddle nose deformity.
How well did you know this?
1
Not at all
2
3
4
5
Perfectly
14
Q

@# 27. A CT scan, obtained to exclude retroperitoneal hematoma in a patient with sustained hypotension for 1 hour after cardiac catheterisation and subsequent cardiac arrest, shows a hypoattenuating renal cortex compared with the medullary enhancement No additional contrast material was given at the time of the CT. Which of the following signs is being described here?
A. Hydronephrotic rim sign
B. Comet tail sign
C. Crescent sign
D. Reverse rim sign
E. Soft-tissue rim sign

A
  1. D. Reverse rim sign

The reverse rim sign refers to a hypoattenuating renal cortex visualised at CT, seen against a background of intact medullary perfusion after contrast material is given. This sign also implies severe derangement of cortical blood flow with development of cortical necrosis.

Cortical necrosis may develop as a consequence of obstetric complications, shock from numerous causes, transfusion reaction or other causes of intravascular haemolysis, toxins and rejection in the transplanted kidney.

The comet tail sign refers to pelvic phleboliths

and the soft-tissue rim sign is related to distal ureteric calculus.

The crescent sign refers to the appearance of concentrated contrast material in collecting tubules, arranged parallel to the margin of a dilated calix, which produces a thin line of contrast material at the edge of the calices, resembling a crescent.

How well did you know this?
1
Not at all
2
3
4
5
Perfectly
15
Q

@# 28. A report is issued stating that there is a periosteal reaction associated with what the radiologist believes is a benign bony lesion. What type of periosteal reaction is most likely to be associated with a benign lesion?
A. Codman
B. Sunburst
C. Hair-on-end
D. Lamination
E. Buttressing

A
  1. E. Buttressing
    Periosteal reaction is a sign of new bone formation. Buttressing indicates a slow-growing process with a single, thick layer of periosteum (i.e., a solid periosteal reaction). It is found in conditions such as hypertrophic pulmonary osteoarthropathy, atherosclerosis and benign tumours.
    More aggressive, faster-growing lesions can cause an interrupted periosteal reaction; lamination (layering) of the periosteum, Codman’s triangles where new bone formation only occurs at the margins of the tumour, sunburst pattern and hair-on-end (this is seen in lesions that invade bone marrow).
    Periosteal reaction is, however, only one of a list of many characteristics of a bone lesion that help determine whether the lesion is biologically active or not.
How well did you know this?
1
Not at all
2
3
4
5
Perfectly
16
Q

@# 29. All of the following are associated with tuberous sclerosis, except
A. Giant cell astrocytoma
B. Subependymal hamartomas
C. Pheochromocytoma
D. Cardiac rhabdomyoma
E. Chylothorax

A
  1. C. Pheochromocytoma
    Tuberous sclerosis is a neuroectodermal disorder, characterised by the classic triad of adenoma sebaceum, seizures and mental retardation. CNS findings include subependymal hamartomas, giant cell astrocytoma (at the foramen of Monro), cortical tubers and heterotopic islands of grey in white matter. Pulmonary involvement include interstitial fibrosis at lung bases, LAM, multiple lung cysts and chylothorax. Cardiac involvement includes rhabdomyoma, aortic aneurysm and congenital cardiomyopathy. Renal involvement includes AML, cysts and RCCs. Ocular lesions include optic nerve glioma and hamartoma. Other visceral abnormalities include splenic tumour, pancreatic and hepatic adenomas.
How well did you know this?
1
Not at all
2
3
4
5
Perfectly
17
Q

@# 30. A 22-month-old boy presents with abdominal pain and is found on US to have an intussusception.
Which of the following statements is false with regard to his initial management and treatment?
A. The patient should be reviewed for signs of peritonitis by a paediatric surgeon.
B. Bowel infarct, peritonitis and perforation are potential complications if the intussusception is left untreated.
C. The patient should be fluid resuscitated prior to attempts at air enema reduction.
D. A plain abdominal radiograph should be obtained prior to air enema reduction to rule out signs of perforation.
E. A successful intussusception reduction is noted when air is seen to fill the small bowel loops

A
  1. D. A plain abdominal radiograph should be obtained prior to air enema reduction to rule
    out signs of perforation.
    The traditional role of plain radiography in the evaluation of children suspected to have intussusception is threefold:
  2. When the clinical suspicion is low, the role of plain radiography is to allow exclusion of intussusception and diagnosis of other pathologic processes that are responsible for the patient’s symptoms.
  3. When the clinical suspicion is high, the role of plain radiography is to allow confirmation of intussusception.
  4. If intussusception is present, the role of plain radiography is to allow exclusion of intestinal obstruction or perforation.
How well did you know this?
1
Not at all
2
3
4
5
Perfectly
18
Q

@# 31. An MRI of the liver with contrast was performed in a 50-year-old woman after a complex mass was identified on US. The mass was reported as suspicious for epithelioid haemangioendothelioma. All of the following are expected features of this lesion, except
A. Vascular origin
B. Female predominance
C. Typical presentation of multifocal nodules
D. Avid arterial phase enhancement
E. Capsular retraction

A
  1. D. Avid arterial-phase enhancement
    HEH is a low-grade primary malignancy of the hepatic vasculature with an incidence of less than
    0.1 per 100,000 in the general population per year. Because its occurrence is so rare, the patient demographics cannot be accurately assessed.
    However, demographic data from case review studies indicate an age range of 25-58 years (average 43.5 years) and a slight female predominance (male to female ratio 2:3). Typical manifestations of HEH include non focal abdominal pain, jaundice and hepatosplenomegaly, but HEH has also been discovered incidentally in asymptomatic patients. Budd-Chiari syndrome can develop if the tumour invades the hepatic veins.
    Alternating high- and low-signal-intensity rings on T2-weighted and contrast-enhanced T1 weighted images have been characterised as a ‘target’ sign. The tumours typically exhibit central signal hypointensity on unenhanced T1-weighted images and a multilayered target-like appearance with prominent rim like enhancement on contrast-enhanced T1-weighted images.
    The presence of capsular retraction in subcapsular hepatic tumours, along with a target-like appearance of the tumours on contrast enhanced CT and MR images, is suggestive but not necessarily indicative of HEH. This constellation of findings can be found in peripheral cholangiocarcinoma, confluent foci of hepatic fibrosis (usually in the setting of advanced cirrhosis), previously treated primary or metastatic HCCs and large atypical cavernous haemangiomas. However, the absence of cirrhosis in a patient with multiple liver masses that coalesce over time to form larger masses, in combination with other characteristic imaging findings of HEH, increases the likelihood of this diagnosis.
How well did you know this?
1
Not at all
2
3
4
5
Perfectly
19
Q

@# 33. Which of the following is incorrect?
A. Spoke wheel sign - Renal oncocytoma
B. Pear-shaped bladder sign - Pelvis lipomatosis
C. Pie-in-the-sky bladder sign - Bilateral psoas hypertrophy
D. Spaghetti sign - Haematuria
E. Balloon-on-a-string sign - Hydronephrosis

A
  1. C. Pie-in-the-sky bladder sign Bilateral psoas hypertrophy

The spoked wheel description was applied to the angiographic appearance of the vascular pattern seen in some oncocytomas. Centripetal ‘spoke’ vessels arising from a peripheral ‘rim’ vessel were initially thought to be characteristic of this tumour. However, the pattern is now known to be non-specific, and a similar vascular arrangement has been described with renal cell carcinoma.

The normal round shape of the opacified bladder may assume a pear or tear drop shape when it is symmetrically compressed in the pelvis by an extrinsic process. The differential diagnosis includes pelvic fluid (haematoma, lymphocele, urinoma or abscess), pelvic lipomatosis, vascular dilatation (aneurysm or collateral vessel development), symmetric lymph node enlargement and psoas muscle hypertrophy.

A pie-in-the-sky bladder may be seen with pelvic trauma. The sign refers to the high position of the opacified bladder within the pelvis at imaging and implies the presence of a large pelvic haematoma; it should raise concern for an associated urethral injury.

A linear filling defect within the bladder may result from extrusion of a blood clot from the ureter, which has acted as a mould, in gross haematuria. This spaghetti sign implies that the bleeding source is above the bladder.

Balloon-on-a-string sign refers to the appearance of a high and somewhat eccentric exit point of the ureter from a dilated renal pelvis and is a typical finding of ureteropelvic junction obstruction.

How well did you know this?
1
Not at all
2
3
4
5
Perfectly
20
Q

@# 34. A young adult man presents with a painful left wrist following a fall from height. A lateral view of the wrist shows loss of co linearity of the radius/lunate/capitate axis with the capitate displaced dorsally. What injury is this constellation of findings compatible with?
A. Midcarpal dislocation
B. Perilunate dislocation
C. Lunate dislocation
D. Volar intercalated segmental instability
E. Dorsal intercalated segmental instability

A
  1. B. Perilunate dislocation

Lunate and perilunate dislocations and fracture dislocations may be easily overlooked on plain films and should be actively sought and excluded given an appropriate clinical history.

On the AP view, the lines along the proximal poles of both the proximal and distal carpal rows should be
smooth and uninterrupted.

The lunate should appear trapezoid and not triangular in shape (AP radiograph), which may indicate dislocation and rotation.

On the lateral view, the distal radius, lunate and capitate should be in alignment.

Volar loss of lunate alignment is seen in lunate dislocation,

and dorsal loss of capitate alignment is seen in perilunate dislocation.

In addition, the scapholunate (normal 30-60 degrees)

and the capitolunate (normal 0-30 degrees) should be assessed for evidence of intercalated segmental instability.

How well did you know this?
1
Not at all
2
3
4
5
Perfectly
21
Q

@# 35. All the following are true of venous angioma, except
A. It can be considered normal.
B. It contains small arterial channels.
C. It is associated with cavernomas.
D. It shows a spoked wheel configuration on imaging.
E. It normally has a large draining vein but no feeding arteries.

A
  1. B. It contains small arterial channels
    Developmental venous anomalies (DVAs), also called venous angiomas, are variations of the normal veins that are necessary for the drainage of white and grey matter. They are associated with other vascular malformations like cavernoma. On contrast-enhanced CT, the venous collector of the DVA is readily detectable as a linear or curvilinear focus of enhancement, typically coursing from the deep white matter to a cortical or a deep vein or to the dural sinus. On MRI, DVAs typically have a trans-hemispheric flow void, on both T1-weighted and T2-weighted images. The collector vein is detected as a linear or small, round, signal-void structure on all sequences and is shown most clearly on T2 weighted imaging. On contrast-enhanced MRI, the cluster of veins in DVAs has a spoked wheel appearance; the veins are small at the periphery and gradually enlarge as they approach a central draining vein. This appearance has been referred to as caput medusae (or the ‘head of Medusa’).
How well did you know this?
1
Not at all
2
3
4
5
Perfectly
22
Q

@# 36. A 5-year old boy, who underwent a bone marrow transplantation 1 week earlier for an underlying haematological malignancy, presents with shortness of breath, cough and chest tightness.
Which of the following complications would be least likely?
A. Lymphoid interstitial pneumonitis
B. Pulmonary oedema
C. Pulmonary embolism
D. Infective interstitial pneumonitis from CMV
E. Idiopathic interstitial pneumonitis

A
  1. A. Lymphoid interstitial pneumonitis
    Early complications complications that occur at any point from the time of transplantation to the end of the early post-engraftment period - include interstitial pneumonitis (infective and non- infective types), infection, oedema, haemorrhage, thromboembolism and calcification. Bronchiolitis obliterans with organising pneumonia (BOOP) is a rare complication that may occur during the early or late post-engraftment period.
    Late pulmonary complications - complications that occur more than 100 days after engraftment - include chronic GVH disease, infections, bronchiolitis obliterans, fibrosis and lymphoid interstitial pneumonitis.
How well did you know this?
1
Not at all
2
3
4
5
Perfectly
23
Q

@# 37. A 56-year-old woman presented to her GP with pain and palpable mass in the right upper quadrant. She was sent for US, which showed a large heterogeneous mass in the right lobe of the liver. A contrast-enhanced MRI of the liver is urgently organised for further characterisation. The lesion is reported as consistent with fibrolamellar carcinoma.
All of the following are expected findings, except
A. Central scar low on T1W and T2W images.
B. Solitary mass is the usual presentation.
C. Favorable prognosis.
I). Rare pathological subtype of hepatocellular carcinoma.
E. Background cirrhotic change to the liver parenchyma.

A
  1. E. Background cirrhotic change to the liver parenchyma

Fibrolamellar HCC is a rare hepatic neoplasm that accounts for less than 1% of HCC in the United States. This tumour has unique clinicopathological features that are significantly different compared with conventional HCC.

Fibrolamellar HCC predominantly occurs in young individuals.

On MRI, fibrolamellar HCC is usually hypointense on T1-weighted images and hyperintense on T2 weighted images.

The fibrous central scar is typically hypointense on both Tl weighted and T2-weighted images. This feature can help to distinguish fibrolamellar HCC from FNH because the central scar in the latter is predominantly T2 hyperintense.

The presence of intralesional fat has not been reported in fibrolamellar HCC. Calcification may be difficult to identify on MRI.

Gadolinium contrast enhancement characteristics of fibrolamellar HCC mimic the patterns seen on CT showing marked heterogeneous contrast enhancement on the arterial phase and becoming isointense or hypointense on the portal venous and delayed phase.

Some authors have reported that fibrolamellar HCC does not retain hepatobiliary-specific contrast agents, such as gadoxetate disodium and gadobenate dimeglumine, on the hepatobiliary phase, which potentially may be useful in differentiating fibrolamellar HCC from FNH.

How well did you know this?
1
Not at all
2
3
4
5
Perfectly
24
Q

@# 38. A 37-year-old woman involved in a frontal car collision and collapse at the scene of incident was brought to the A&E department and sent for an emergency whole-body CT. All of the following are correct regarding blunt cardiac trauma, except
A. Cardiac concussion results in abnormal cardiac enzymes.
B. Traumatic pericardial rupture resulting from blunt chest trauma is rare.
C. Cardiac herniation is a serious complication of pericardial rupture.
D. Traumatic ventricular septal defects affect the muscular portion.
E. Myocardial contusion is associated with cardiac tamponade.

A
  1. A. Cardiac concussion results in abnormal cardiac enzymes.

Blunt cardiac injury (BCI) is the most common type of cardiac injury after blunt thoracic trauma. In cardiac concussion, the mildest form of cardiac injury, there is no myocardial cell damage or elevated enzyme levels. Cardiac contusion can present as bilateral cardiogenic pulmonary oedema and elevated cardiac enzymes. Echocardiography shows increased myocardial echogenicity and focal systolic hypokinesia, and it is useful in diagnosing other traumatic cardiac injuries that are commonly associated with myocardial contusion, such as pericardial effusion, tamponade, traumatic ventricular septal defect and valvular injury. Typically, traumatic ventricular septal defects occur in the muscular portion of the interventricular septum, near the cardiac apex.
Haemopericardium is commonly associated with cardiac rupture. Tamponade resulting from ventricular rupture is often fatal; however, bleeding from lower pressure atria may be survivable. Besides cardiac chamber rupture, traumatic haemopericardium may also result from aortic root injury, myocardial contusion and coronary artery laceration.
Traumatic pericardial rupture resulting from blunt chest trauma is rare. Tearing may involve either the pleuropericardium or the diaphragmatic pericardium. Cardiac herniation is a serious complication of pericardial rupture.

How well did you know this?
1
Not at all
2
3
4
5
Perfectly
25
Q

@# 39. An 88-year-old man with obstructive uropathy and hard nodular prostate gland on digital rectal examination was being further investigated for his prostate cancer. Which one of the following magnetic resonance spectroscopic imaging metabolic peaks is expected in a patient with prostate cancer?
A. High citrate peak, low choline/creatine peak
B. High citrate peak, high choline/creatine peak
C. Low citrate peak, high choline/creatine peak
D. Low citrate peak, low choline/creatine peak
E. Low citrate peak, normal choline/creatine peak

A
  1. C. Low citrate peak, high choline/creatine peak

MRS is a modality that provides information about the cellular metabolites within the prostate gland; it displays the relative concentrations of key chemical constituents, such as citrate, choline and creatine. The normal prostate gland contains low levels of choline and high levels of citrate, whereas prostate cancers have increased levels of choline and decreased levels of citrate. The high choline levels in prostate tumours are related to increased cell turnover. In prostate cancers, the levels of zinc in secretary epithelial cells are lower, which eventually leads to diminished amounts of citrate. Thus, the ratio of choline to citrate is an index of malignancy. The ratio of choline (Cho) plus creatine (Cr) to citrate (Cit) ([Cho + Cr]/Cit) has been widely studied and cut-off values have been suggested for detection of cancer.
Besides analysis of choline, citrate and creatine content, newer image acquisition and analysis software may enable evaluation of other metabolites, such as polyamine peaks, which have also been associated with prostate cancers. MR spectroscopy of tissue samples revealed that levels of polyamines (putrescine and spermine) were reduced in prostate cancer compared with the levels in benign tissue.

How well did you know this?
1
Not at all
2
3
4
5
Perfectly
26
Q

@# 40. A 42-year old woman with knee joint stiffness, pain and reduced mobility is investigated with a plain X-ray, which reveals extensive soft-tissue swelling around the joint with
large periarticular erosions. There is no evidence of calcification and bone density is maintained. MRI shows low-signal foci within the soft-tissue mass. What is the likely diagnosis?
A. Pigmented villonodular synovitis
B. Haemophilia
C. Synovial sarcoma
D. Behcet’s syndrome
E. Psoriatic arthropathy

A
  1. A. Pigmented villonodular synovitis.
    Radiographs may be normal or may reveal periarticular soft-tissue swelling. Visible calcifications are extremely unusual. The joint space is normal in width until late in the course of the disease, and juxta-articular osteoporosis is absent or mild. Bone erosions with sclerotic margins may be present on both sides of the affected joints, particularly in joints with a tight capsule, such as the hip. On MRI, the synovial lesions have low to intermediate signal intensity relative to that of muscle on T1 weighted images and low signal intensity on T2-weighted images, caused by the magnetic susceptibility effect produced by haemosiderin at the periphery of the lesions.
    The ‘black synovium’ appearance can also be seen with amyloid deposition or recurrent intra- articular bleeding (e.g., in haemophiliac arthropathy).
    Synovial sarcoma presents as a solitary calcified mass outside the joint.
How well did you know this?
1
Not at all
2
3
4
5
Perfectly
27
Q

@# 41. A 40-year-old woman with progressive difficulty in walking and difficulty in holding objects reveals a large burn over the dorsum of her right hand on clinical examination. Sagittal MRI of the cervical spine shows a long segment lesion in the cervical and upper thoracic spinal cord with low signal on T1W images and high signal on T2W images without
any change on post-contrast images. What is the diagnosis?
A. Ependymoma
B. Syringomyelia
C. Astrocytoma
D. Haemangioblastoma
E. AVM

A
  1. B. Syringomyelia
    Syringomyelia refers to any cavity within the substance of the spinal cord that may communicate with the central canal, usually extending over several vertebral segments. MRI shows a cystic area with low signal on T1-weighted and increased signal intensity on T2-weighted images. CSF flow void (low signal on T2-weighted images) is often seen in the cavity from pulsation. The cord may show’ enlargement.
    Post-traumatic, post-inflammatory or idiopathic syringomyelia show no abnormal contrast enhancement post-administration of gadolinium. Tumour-associated syrinx may show abnormal enhancement.
How well did you know this?
1
Not at all
2
3
4
5
Perfectly
28
Q

@# 42. A chest CT is performed on a 10-year-old boy with known underlying chronic lung changes. Current imaging demonstrates laevocardia with areas of air trapping, bronchial dilatation and bronchial wall thickening, which were also present on previous imaging from 1 year earlier. There are no significantly enlarged mediastinal lymph nodes.
Which of the following differential diagnoses would be least likely to account for the underlying changes?
A. Cystic fibrosis
B. IgA deficiency
C. Juvenile dermatomyositis
D. Primary ciliary dyskinesia
E. Recurrent pneumonias

A
  1. C. Juvenile dermatomyositis

All other options would cause bronchiectasis, apart from juvenile dermatomyositis (JDM). JDM is a multisystem autoimmune disease of unknown aetiology that results from inflammation of the small vessels of the muscles, skin, gastrointestinal tract and other organs. It accounts for 85% of cases of paediatric inflammatory myopathy. JDM affects children with an incidence of two or three cases per million children per year. Clinical characteristics of JDM include symmetric proximal muscle weakness, inflammatory cutaneous lesions and Gottron papules (erythematous scaly lesions over the metacarpophalangeal and/or interphalangeal joints), heliotrope (violaceous hue over the eyelids), periorbital oedema, malar erythema, periungual telangiectasia and erythematous scaly rashes over the neck, upper back and extensor surfaces of the extremities.

How well did you know this?
1
Not at all
2
3
4
5
Perfectly
29
Q

@# 44. A 43-year-old woman with known diagnosis of tuberous sclerosis presented to her GP with increasing right upper-abdominal pain. Although the LFT was normal, the GP organised a liver MR in search of angiomyolipomas in the liver. All of the following arc MR features of angiomyolipomas of the liver, except
A. A well-defined mass.
B. Moderately hyperintense signal intensity on T1W images.
C. Moderately hyperintense signal intensity on T2W images.
D. Loss of signal on fat-suppressed sequences.
E. Usually shows late arterial enhancement on the post-contrast sequence.

A
  1. E. Usually shows late arterial enhancement on the post-contrast sequence.

Angiomyolipoma (AML) is a benign, unencapsulated mesenchymal tumour that is composed of
varying proportions of three elements: smooth muscle cells, thick-walled blood vessels and mature adipose tissue. AML occurs more commonly in the kidneys; hepatic involvement is rare. In contrast to renal AML, which is associated with tuberous sclerosis in 20% of patients, hepatic AML is associated with tuberous sclerosis in only 6%. AML can be histologically classified on the basis of fat content into mixed, lipomatous (70% fat), myomatous (10% fat) and angiomatous types. US, CT and MR imaging typically demonstrate the fat component and prominent central vessels. At US, AML may be highly echogenic and is then indistinguishable from haemangiomas.
Frequently, AML has a high fat content, with high signal intensity on T1-weighted images and a significant drop in signal intensity on fat-suppressed images.

AML demonstrates early intense contrast enhancement that peaks later than that of a HCC. Dynamic contrast-enhanced CT or MR images obtained during the early phase of enhancement may be useful in discriminating between AML and fat-containing HCC. The fatty areas of AMLs are well vascularised and enhance early, whereas steatotic foci in HCC are relatively avascular and have less contrast enhancement. However, unlike renal AML, 50% of hepatic AMLs lack considerable fat content. Because of this variable fat content, it is difficult to accurately distinguish AML from other hepatic tumours.

How well did you know this?
1
Not at all
2
3
4
5
Perfectly
30
Q

@# 45. With regard to the progression of pulmonary consolidation on CXR, all of the following options are true except
A. Lung contusion appears in 6 hours and clears in 3-7 days.
B. Aspiration appears in minutes and clears in 24-48 hours unless infected.
C. Lung infarction due to pulmonary embolism manifests after 3-5 days and clears in approximately 3 weeks.
D. Fat embolism appears in 6 hours and clears in 7-10 days.
E. ARDS with difFuse alveolar damage appears after 24-48 hours and clears in 4-6 weeks.

A
  1. D. Fat embolism appears in 6 hours and clears in 7-10 days.

Lung contusion turns out to be the most common manifestation of blunt trauma to the chest. Contusions become visible on chest radiographs within 6 hours post-trauma and resolve within 3-7 days. Massive contusion can lead to pulmonary oedema and diffuse alveolar damage (DAD).

Aspiration appears within minutes and clears in 24-48 hours unless it triggers an infection with subsequent pneumonia.

Pneumonia appears after 5-7 days and clears in 2-4 weeks.

Lung infarction due to pulmonary embolism manifests after 3-5 days and clears in approximately 3 weeks.

Fat embolism appears after 24-48 hours and clears in 7-10 days.

ARDS with diffuse alveolar damage appears after 24-48 hours and dears in 4-6 weeks.

How well did you know this?
1
Not at all
2
3
4
5
Perfectly
31
Q

@# 46. An 88-year-old man with obstructive uropathy and hard nodular prostate gland on digital rectal examination is being further investigated for his prostate cancer. lie has been scheduled for a staging MRI of the prostate. Which one of the following is the characteristic imaging finding of prostate cancer on DWI?
A. High on DWT, low on ADC and corresponding low signal intensity on T2W MRI
B. Low on DWI, high on ADC and corresponding low signal intensity on T2W MRI
C. High on DWI, low on ADC and corresponding high signal intensity on T2W MRI
D. Low on DWI, high on ADC and corresponding high signal intensity on T2W MRI
E. High on DWI, low on ADC and corresponding normal signal intensity on T2W MRI

A
  1. A. High on DWI, low on ADC and corresponding low signal intensity on T2W MRI

T2-weighted images are the ‘workhorse’ images for prostate cancer. The glandular peripheral
zone appears high in signal, whereas the central stroma has lower signal intensity on T2-weighted images. Tumours are lower in signal intensity than normal peripheral zone glandular tissue on T2-weighted images. Peripheral zone cancers are usually round or ill-defined. Various conditions, such as prostatitis, haemorrhage, atrophy, benign prostatic hyperplasia (BPH) and post-treatment changes can mimic cancer. Cancers in the central gland are even more challenging to detect than peripheral zone cancers, because the signal characteristics of the normal and hypertrophic central gland arc usually similar to those of the tumour on T2-weighted images. Prostate cancers often include tightly packed glandular elements with increased cellular density and diminished extracellular spaces, which can be detected as high-signal-intensity foci on raw DW MRI (restricted diffusion) but are low in signal on apparent diffusion coefficient (ADC) maps. ADC maps reflect the amount of diffusion present - the lower the diffusion, the darker the lesion.

32
Q

@# 47. A 50-year-old patient presents with shortness of breath to his GP. Following abnormalities on the chest radiograph, a CT is performed, demonstrating bilateral hilar and right paratracheal lymphadenopathy along with coarse reticulations and nodular thickening along the fissures and bronchovascular bundles. Elevated ACE serum levels are noted.
What is not a typical musculoskeletal manifestation of this disease?
A. Lacy lytic lesions in the phalanges of the hands
B. Polyarthralgia with tenosynovitis
C. Multiple bony lesions with high T2W and intermediate T1W signal characteristics
D. Soft-tissue calcifications
E. Dactylitis

A
  1. D. Soft tissue calcifications
    Acute arthritis may be the first manifestation of sarcoidosis. It is mainly oligoarticular but occasionally polyarticular, and rarely monoarticular. Ankles are the most commonly involved joints. Enthesitis, tendinosis and tenosynovitis are well recognised, especially around the ankle. The triad of acute arthritis, bilateral hilar adenopathy and erythema nodosum is known as Lofgren syndrome. Chronic arthritis tends to involve the shoulders, hands, wrists, ankles and knees. Dactylitis can be seen; it is very similar to that seen in patients with psoriatic arthritis.
    Osseous involvement causes cystic, reticular or destructive lesions involving mainly the hands and feet (lace-like pattern) but can affect the skull, ribs, sternum, vertebrae, nasal bones, pelvis, tibia and femur as well. MRI shows multiple bony lesions with high T2-weighted and intermediate T1-weighted signal, although diffuse involvement is also recognised. Sarcoid myopathy can also he acute and mimic polymyositis.
33
Q

@# 50. A 73-year-old woman with a known splenic lesion underwent a CT of the chest, which demonstrated multiple pulmonary nodules with surrounding halos. A follow-up CT chest done a few weeks later showed cavitations developing in some of the nodules and a right pneumothorax. The most likely diagnosis is
A. Lymphoma
B. Hydatid disease
C. Metastatic angiosarcoma
D. Sarcoidosis
E. Bronchoalveolar carcinoma

A
  1. C. Metastatic angiosarcoma

Squamous cell carcinomas are regarded as the most common type of cavitating metastases, although adenocarcinomas are also implicated. Metastatic sarcomas can also cavitate, and a pneumothorax can be complicated.
Angiosarcoma represents the most common non-lymphoid malignant tumour of the spleen. Imaging reveals splenomegaly and an ill-defined heterogeneous mass.

34
Q

@# 51. A 77-year-old man with obstructive uropathy and TRUS biopsy-proven prostate cancer was scheduled to have an MRI for local staging of the known prostate cancer. Which one of the following statements regarding identification of prostate cancer by dynamic contrast enhancement (DCE) is true?
A. Fast enhancement and slow washout suggest prostate cancer.
B. Slow enhancement peak and slow washout suggest prostate cancer.
C. Slow enhancement and fast washout are highly suggestive of prostate cancer.
D. Small prostate tumours are detected easily.
E. The higher the tumour grade, the greater the grade of enhancement.

A
  1. E. The higher the tumour grade, the greater the grade of enhancement

DCE-MRI evaluates the vascularity of tumours by providing quantitative kinetic parameters that reflect the flow of blood and the permeability of the vessels. DCE-MRI increases the specificity of prostate tumour detection. Tumours show early enhancement and early washout of the contrast agent - the higher the tumour grade, the higher would be the permeability parameters (Ktrans - wash-in, Kcp washout). A disadvantage of DCE-MRI is that small, low-grade tumours may not demonstrate abnormal enhancement. Furthermore, abnormal enhancement patterns can also be seen in patients with BPH, which can make assessment of the central gland difficult. However, in the glandular peripheral zone and anterior gland, DCE-MRI can be helpful in identifying lesions that are not suspected on T2-weighted images. The degree of enhancement on DCE-MRI correlates positively with tumour grade.

35
Q

@# 52. A 40 year old man presents to his general practitioner with intermittent left leg pain and stiffness of 5 years duration. Plain radiograph reveals eccentric and irregular sclerosis along the medial aspect of the distal femur, crossing the joint into the adjacent tibia. Cortical new bone formation along the outer aspect of the cortex is also seen, with an appearance similar to flowing wax. In addition, there are adjacent ossified soft-tissue masses. Which of the following is the most likely diagnosis?
A. Melorheostosis
B. Osteopoikilosis
C. Fibrous dysplasia
D. Osteoarthritis
E. Osteopetrosis

A
  1. A. Melorheostosis

Melorheostosis is a very rare non-hereditary disease of unknown aetiology. Patients can present with pain and restricted movements of the joint, but the disease is usually asymptomatic and is discovered incidentally. Some patients may have thickening and fibrosis of overlying skin, resembling scleroderma or joint contractures. Radiographic features include dense irregular bone running down the cortex of a tubular long bone, which can cross the joint. This has the appearance of flowing/molten wax and is usually limited to one side of cortex, although the internal or external cortex may be affected. The lower limbs are the most commonly affected. Other notable features are discrepant leg length on the affected side and the presence of adjacent ossified soft-tissue masses (27%).

36
Q

@# 53. A 37-year-old woman presented to the A&E department with acute worsening of arm weakness and pins and needles at the C4/5 levels. MRI of the spine revealed an elongated area of high T2W signal change, which showed variable contrast enhancement. There was no
associated expansion of the cord, and CSF obtained by LP had earlier shown oligoclonal immunoglobulin bands. What is the likely diagnosis?
A. Lyme disease
B. Multiple sclerosis
C. Sarcoidosis
D. LCH
E. Lymphoma

A
  1. B. Multiple sclerosis
    The spinal cord is known to be frequently involved in MS or in combination with lesions in the brain; as many as 25% of cases have been found to involve only the spinal cord. Most spinal cord lesions occur in the cervical cord. Cervical spinal MRI is particularly useful when the patient is suspected to have MS and brain lesions are absent. On T2-weighted spinal imaging, most plaques are peripherally located (commonly dorsolateral) and less than two vertebral body segments in length. The lesions tend to be multifocal and present as well-circumscribed foci with an increased T2 signal intensity. With acute spinal cord lesions, enhancement is frequently seen. Spinal cord atrophy, which may reflect axonal loss, may also be observed and is believed to be an important element in disability. Though rarely seen in other diseases, asymptomatic lesions of the spinal cord can be present in MS and may help lead to the correct diagnosis.
    Unlike its applicability to the brain, FLAIR imaging appears unreliable in the detection of MS lesions in the spinal cord. The short inversion time inversion-recovery (STIR) sequence has been found to be sensitive for lesion detection, particularly diffuse lesions.
37
Q

@# 54. A 9-year-old female patient is incidentally found to have a pelvic mass on US. The features are thought to represent those of a mature cystic teratoma (dermoid cyst).
Which of the following complications relating to this diagnosis is the least likely?
A. Malignant transformation
B. Associated ovarian torsion
C. Rupture of the lesion
D. Autoimmune haemolytic anaemia
E. Bleeding

A
  1. D. Autoimmune haemolytic anaemia

Mature cystic teratoma can be associated with complications from rupture, malignant degeneration or (most commonly) torsion. Mature cystic teratomas affected by torsion are larger than average (mean diameter 11 cm vs. 6 cm); this enlargement could be the result of the torsion rather than the cause of it
Most mature cystic teratomas can be diagnosed at US. However, the US diagnosis is complicated by the fact that these tumours may have a variety of appearances. Three manifestations occur most commonly. The most common manifestation is a cystic lesion with a densely echogenic tubercle (Rokitansky nodule) projecting into the cyst lumen. The second manifestation is a diffusely or partially echogenic mass with the echogenic area usually demonstrating sound attenuation owing to sebaceous material and hair within the cyst cavity.

38
Q

@# 55. An 11-year-old girl is currently under investigation for an adnexal mass recently found on a US scan. Which of the following statements regarding paediatric ovarian tumours is least accurate?
A. Gonadoblastomas are associated with dysgenetic gonads.
B. Sertoli-Leydig cell tumours may present with virilizing symptoms.
C. Mature cystic teratomas are the most common ovarian tumours in children.
D. Immature teratomas are associated with elevated serum AFP levels.
E. Elevated CA-125 levels are commonly seen in children with germ cell tumours.

A
  1. E. Elevated CA-125 levels are commonly seen in children with germ cell tumours.

Mature cystic teratoma, often called dermoid cyst when the ectodermal elements predominate, is
the most common ovarian tumour in children and adolescents, accounting for approximately 50% of all paediatric ovarian neoplasms.

CA-125 levels are more frequently elevated in patients with epithelial ovarian cancers than in those with germ cell tumours (GCTs). CA 125 levels should not be routinely obtained in children, particularly in premenarcheal girls, as epithelial ovarian tumours are extremely rare before menarche.

The serum AFP level is elevated in patients with GCTs such as yolk sac tumours, immature teratomas, embryonal carcinomas and mixed GCTs with yolk sac elements.

39
Q

@# 58. An 81-year-old man with progressive worsening hesitancy, dribbling, incomplete voiding and haematuria was found to have a hard nodular prostate gland on digital rectal examination. He is due to have a staging prostatic MRI, which will take place shortly after TRUS biopsy. Which one of the following is characteristic of post-biopsy haemorrhage of the prostate gland on MR imaging?
A. Hyperintense on T1W imaging and hypointense on T2W imaging
B. Hypointense on T1W imaging and hypointense on T2W imaging
C. Hypointense on T1W imaging and hyperintense on T2W imaging
D. Hyperintense on T1W imaging and hyperintense on T2W imaging
E. Hyperintense on T1W imaging and isointense on T2W imaging

A
  1. A. Hyperintense on T1W imaging and hypointense on T2W imaging

T1-weighted images are important in prostate cancer staging, because they show the presence of haemorrhage secondary to a recent biopsy; haemorrhage is almost always hyperintense compared with normal parenchyma on T1-weighted MRI. Haemorrhage typically appears low in signal on T2-weighted MRI and mimics cancer.

40
Q

@# 59. A 52-year-old man has a lower-leg radiograph performed after a fall. No acute bony injury is identified, but the reviewing doctor notices a flame-shaped lucency involving the medulla of the tibia, extending superiorly from the distal subarticular into the diaphysis. He is unsure of the significance of this appearance and seeks the opinion of the radiologist. Which of the following is the most likely diagnosis?
A. Osteomyelitis
B. Giant cell tumour
C. Plasmacytoma
I). Paget disease (acute phase)
E. Paget disease (chronic phase)

A
  1. D. Paget disease (acute phase)

Paget disease is a multifocal skeletal condition whereby bone remodelling becomes abnormal and eventually exaggerated. It typically affects middle-aged to elderly people and has acute and chronic phases with differing radiographic appearances. The acute phase, also known as the active or osteolytic phase, is characterised by aggressive bone resorption secondary to disordered osteoclastic activity. This manifests as lucency within the medulla of the long bones, typically in a flame shape or ‘blade of grass’ configuration. Over time, osteoblastic activity overtakes the osteoclastic resorption, resulting in osteosclerosis, cortical thickening and increased trabeculation. Giant cell tumour is a benign neoplasm of the mature skeleton, seen almost exclusively in those under 30 years of age. Although osteomyelitis can result in bony lucency, it is typically patchy with a moth-eaten texture.

41
Q

@# 61. A 42-year-old woman was found to have a mass in the right lobe of the liver on US and sent for an MRI liver for further characterisation. All of the following are expected MR features of FNH, except
A. High signal intensity of the central scar on T2W images
B. Uniform enhancement of the mass in the arterial phase
C. Lack of capsular enhancement in the arterial phase
D. Hypointense to surrounding liver on the enhanced portal venous phase
E. Maximum intensity of central scar on enhanced delayed phase images

A
  1. D. Hypointense to surrounding liver on the enhanced portal venous phase

Typically, FNH is iso- or hypointense on T1-weighted images, is slightly hyper- or isointense on T2-weighted images and has a hyperintense central scar on T2-weighted images.

FNH shows intense homogeneous enhancement in the arterial phase and enhancement of the central scar in the later phases of gadolinium-enhanced imaging.

FNH does not have a tumour capsule, although the pseudocapsule surrounding some FNH lesions may be quite prominent. The pseudocapsule of FNH results from compression of the surrounding liver parenchyma by the FNH, peri-lesion vessels and inflammatory reaction. The pseudocapsule is usually a few millimetres thick and typically shows high signal intensity on T2-weighted images. The pseudocapsule may show enhancement on delayed contrast-enhanced images.

A central scar is present at imaging in most patients with FNH. The amount of scar tissue within FNH and the size of the central scar may vary. The central scar is typically high in signal intensity on T2-weighted images and low in signal intensity on T1-weighted images. It shows visible enhancement on delayed contrast enhanced images. High signal intensity of the central scar may be caused by the inflammatory reaction around the ductular proliferation as well as the vessels within the septa and central scar. The central scar is not a specific finding of FNH and can be seen in a variety of other focal liver lesions, such as giant haemangiomas.

42
Q

@# 64. A 15-year-old boy, who is a keen footballer, has a radiograph taken of the left knee following a tackle. There is an area of periosteal reaction and irregular speculated periosteum at the posterior aspect of the distal femur just proximal to the physis. He denies any symptoms before the injury. The next imaging should be which one of the following?
A. A comparison radiograph of the contralateral knee
B. Non-contrast CT of the distal femur
C. Contrast-enhanced CT of the distal femur
D. MRI of the distal femur
E. A nuclear medicine bone scan

A
  1. A. A comparison radiograph of the contralateral knee

The appearance is likely to represent post-traumatic cortical desmoid. It is also known as avulsive cortical irregularity or Bufkin lesion. It results from chronic avulsive stress at the femoral region of the medial head of the gastrocnemius. It is usually an incidental finding. One-third of cases are bilateral and a comparison photograph of the contralateral knee is helpful to assess the other side.
If plain films are not helpful, an MRI can be useful to assess soft tissues. Post-traumatic cortical desmoid does not involve soft tissues.

43
Q

@# 65. A 16-year-old boy with learning difficulties presented to the A&E department with increasing frequency of epileptic fits, which he has had since childhood. Bilateral brisk reflexes
were noted clinically. CT of the brain performed to exclude haemorrhage revealed a large heterogeneous lobulated mass replacing the third ventricle with further areas of subependymal calcification. What is the diagnosis?
A. Sturge-Weber syndrome
B. Tuberous sclerosis
C. Arnold-Chiari malformation
D. Neuroblastoma metastasis
E. VHL

A
  1. B. Tuberous sclerosis
    Tuberous sclerosis is a neuroectodermal disorder, characterised by the classic triad of adenoma sebaceum, seizures and mental retardation. CNS findings include subependymal hamartomas, which can calcify (classic calcified subependymal nodules), giant cell astrocytoma (at the foramen of Monro, often replacing the third ventricle), cortical tubers and heterotopic islands of grey in white matter. Pulmonary involvement includes interstitial fibrosis at lung bases, LAM, multiple lung cysts and chylothorax. Cardiac involvement includes rhabdomyoma, aortic aneurysm and congenital cardiomyopathy. Renal involvement includes AML, cysts and RCCs. Ocular lesions include optic nerve glioma and hamartoma. Other visceral abnormalities include splenic tumour and pancreatic and hepatic adenomas.
44
Q

@# 67. A 14-year-old female patient is found on US to have a large pelvic mass comprised of mainly cystic elements. The right ovary is not seen separate to the mass.
With regard to adnexal masses in children, which of the following statements is true?
A. Mature cystic teratomas tend to be bilateral when present.
B. A raised serum AFP level is indicative of choriocarcinoma.
C. The majority of mature cystic teratomas are malignant.
D. The most common malignant germ coll tumour is a dysgerminoma.
E. Yolk sac tumours tend to have a benign clinical course.

A
  1. D. The most common malignant germ cell tumour is a dysgerminoma.

Germ cell tumours are the most common type of ovarian neoplasm, with one third of these being malignant in children and adolescents. A dysgerminoma is the most common malignant germ cell tumour and is associated with gonadal dysgenesis.
The majority of germ cell tumours are mature cystic teratomas (also known as dermoid cysts), which account for 50% of all paediatric ovarian neoplasms. These are bilateral in 10% of cases.

45
Q

@# 68. A 33-year-old lady with a history of right upper-quadrant pain and deranged LFTs is sent for an ultrasound, which shows a large cystic lesion in the right liver lobe. Blood tests also show’ eosinophilia. Which of the following statements regarding the imaging of Hydatid disease is false?
A. US findings are variable and range from purely cystic to solid-appearing pseudotumours.
13. Daughter cysts are frequently seen.
C. Cysts usually appear well defined on CT.
D. Coarse wall calcification may be present,
E. CT imaging best demonstrates the pericyst, matrix and daughter cysts.

A
  1. E. CT imaging best demonstrates the pericyst, matrix and daughter cysts.

Hydatid disease is a severe and common parasitic disease that is endemic to the Mediterranean basin and other sheep raising areas. Humans become infected by ingesting the eggs of the tapeworm Echinococcus granulosus, either by eating contaminated food or from contact with dogs. The ingested embryos invade the intestinal mucosal wall and proceed to the liver via the portal venous system. Although the liver filters out most of these embryos, those that are not destroyed become hepatic hydatid cysts.
US findings are variable and range from purely cystic to solid-appearing pseudotumours. Wavy bands of delaminated endocyst (water lily sign) may be noted internally. Daughter cysts, sometimes surrounded by echogenic debris (matrix), are frequently seen. Calcifications, varying from tiny to massive, are often present peripherally.
At CT, a hydatid cyst usually appears as a well-defined, hypoattenuating lesion with a distinguishable wall. Coarse wall calcifications are present in 50% of cases, and daughter cysts are identified in approximately 75%. Because of its superb contrast resolution, MR imaging best demonstrates the pericyst, the matrix or hydatid sand (debris consisting of freed scolices) and the daughter cysts.

The pericyst is seen as a hypointense rim on both T1-weighted and T2-weighted images because of its fibrous composition and the presence of calcifications. The hydatid matrix appears hypointense on T1-weighted images and markedly hyperintense on T2-weighted images; when present, daughter cysts are hypointense relative to the matrix on both Tl - weighted and T2-weighted images.

46
Q

@# 70. A 37-year-old woman in her third trimester presented to the labour ward with acute onset of pain in the lower central abdomen. She had had two previous caesarean sections and an appendectomy with unremarkable recovery. On clinical examination, there was definite tenderness at the site of a previous caesarean scar. The best imaging modality for evaluation of caesarean section scar dehiscence is
A. CT
B. Transabdominal US
C. MRI
D. Angiography
E. Transvaginal US

A
  1. C. MRI

Uterine dehiscence is characterised by incomplete rupture of the uterine wall, usually involving the endometrium and myometrium but with an intact overlying serosal layer. MR imaging may be better than CT in evaluating for uterine dehiscence because of its multiplanar capability and greater soft tissue contrast and its ability to help identify an intact serosal layer.

47
Q

@# 71. A 23-year-old man presents with an acute injury of his knee sustained during a friendly football match following a rough tackle. The patient reports having heard a pop and the knee started swelling immediately. He is unable to weight-bear and had to be carried off the field. An MRI is organised with high suspicion of ACL injury. All of the following are indirect signs of ACL injury, except
A. Bone bruising in lateral tibiofemoral compartment
B. Deep lateral femoral sulcus >1.5 mm
C. Uncovering of the posterior horn of the lateral meniscus
D. Posterior translation of the tibia
E. PCL bowing <105 degrees

A
  1. D. Posterior translation of tibia

An ACL injury results in anterior translation of the tibia. The ‘kissing contusions’ from a pivot shift mechanism of injury results in marrow oedema in the posterolateral region of the tibial plateau and the mid portion of the lateral femoral condyle. A deep lateral femoral sulcus, more than 1.5 mm, may be appreciated on plain film. Uncovering of the posterior horn of the lateral meniscus is associated with the anterior translation of the tibia. The PCL tends to buckle after an ACL injury resulting in a reduced PCL angle less than 105 degrees.

48
Q

@# 72. MRI brain images of a 42-year- old man diagnosed with HIV showed areas of cloud-like high signal on T2W images in the periventricular area bilaterally. All of the following are true regarding CNS infections in HIV, except
A. High signal in the periventricular area suggest HIV subacute encephalitis.
B. Subacute encephalitis caused by HIV and CMV have similar appearances.
C. Basal meningitis suggests neurosyphilis.
D. Cerebral infarcts can occur with neurosyphilis in HIV patients.
E. CMV infection can manifest as ventriculoencephalitis

A
  1. C. Basal meningitis suggests neurosyphilis.

The finding of diffuse cerebral atrophy in HIV can be accompanied by diffuse, confluent, ill-defined areas of abnormally increased signal intensity on T2-weighted MR images of the periventricular white matter. No enhancement is noted. These findings were initially attributed to CMV infection, as autopsy studies have shown that a high percentage of AIDS patients had concomitant CMV infection of the CNS. It was subsequently recognised that HIV is neurotrophic and directly infects the CNS; these global abnormalities were then attributed to a subacute encephalitis caused by HIV. It now seems likely that both HIV and CMV can cause subacute encephalitis and encephalopathy; these conditions have an identical non-specific imaging appearance. CMV encephalopathy appears to manifest itself late in the illness, whereas HIV rather than HIV dementia (although usually presenting late) can occasionally be the AIDS-defining illness.
In addition to subacute encephalitis, CMV infection can manifest as a more virulent ventriculoencephalitis. This is most often seen in concert with CMV infection elsewhere in the body (such as CMV retinitis) and is usually a late manifestation of AIDS. The infection appears to begin in the ependymal or subependymal region and spread into the adjacent brain. The ventricles are enlarged, and hypoattenuation on CT or increased signal intensity on T2-weighted MRI is noted in the periventricular white matter. Ill-defined periventricular enhancement can also be seen following injection of contrast medium.

Meningitis - particularly basilar meningitis - and hydrocephalus occur often in the setting of neurotuberculosis. Meningitis manifests on CT scans and MR images as prominent enhancement and thickening of the meninges and it can be associated with infarcts secondary to vascular stenosis or occlusion.

Neurosyphilis, which is being diagnosed with increased frequency among patients with AIDS, can cause infarcts secondary to arteritis.

49
Q

@# 73. A 74-year-old woman presented with post-operative gastric cancer, at 4 days after onset of fever, cough and sputum. CT was organised for characterisation. All of the following are true regarding MRSA and MSS A pneumonia on CT chest, except
A. Lung nodules are more common in MSS A.
B. Effusion is more common in MRSA.
C. There is no significant difference in zonal distribution between MSSA and MRSA pneumonia.
D. Hilar lymph nodes are more common in MRSA.
E. Consolidation is seen in both groups.

A
  1. D. Hilar lymph nodes are more common in MRSA

Radiological findings of MRSA pneumonia reported in the current literature are limited to descriptions of necrotising pneumonia.

The frequencies of bronchial wall thickening and centrilobular nodules were significantly higher in patients with MSSA than MRSA pneumonia.

Moreover, the frequency of centrilobular nodules with a tree-in-bud pattern was significantly higher in patients with MSSA than MRSA pneumonia.

There were no significant differences in other CT findings including ground-glass opacity and consolidation between the two groups.

Ground-glass change and bronchial wall thickening were the two most common CT findings in both groups.

There were no significant differences in zonal distributions between the two groups. Both groups showed unilateral or bilateral opacities and predominantly upper zone changes.

The frequency of pleural effusion was significantly higher in patients with MRSA than in those with MSSA pneumonia. There was no difference in hilar or mediastinal lymphadenopathy.

50
Q

@# 75. A 42-year-old man with epigastric pain and upper-abdominal fullness was sent for US assessment. No gallstones were observed and bile ducts were normal. Multiple tiny liver lesions were noted and an MRI was advised for characterisation. MRI was reported as biliary hamartomas (von Meyenburg complexes, VMC) of the liver and the patient was reassured. Which one of the following statements concerning VMC is false?
A. They may be solitary or multiple.
B. They are benign cystic biliary malformations.
C. They are high signal intensity on T1W MRI.
D. They usually are smaller than 1 cm in diameter.
E. They do not show perilesional enhancement on MRI.

A
  1. C. They are high signal intensity on T1W MR images.

Biliary hamartomas, also known as biliary microhamartomas or von Meyenburg complex, are composed of one or more dilated duct-like structures lined by biliary epithelium and accompanied by a variable amount of fibrous stroma. Biliary hamartomas arc typically multiple round or irregular focal lesions of nearly uniform size (up to 15 mm) scattered throughout the liver. ‘These lesions are often discovered incidentally, and if the patient has a primary neoplasm they can be mistaken for metastatic disease. The lesions are hypoattenuating at CT, hypointense at T1-weighted MR imaging and hyperintense at T2-weighted imaging. If the echo time is increased at T2-weighted imaging, the signal intensity of these lesions increases further and approaches that
of cerebrospinal fluid. The lesions do not usually show contrast enhancement, although a peripheral enhancing rim has been described.
The differential diagnoses for biliary hamartomas include metastatic disease and simple hepatic cysts. Biliary hamartomas are relatively uniform in size, whereas metastatic lesions are usually more heterogeneous in size and in attenuation or signal intensity. Compared with biliary hamartomas, hepatic cysts are rarely as uniformly small or numerous, whereas the cysts in autosomal dominant polycystic disease are usually larger and more numerous.

51
Q

@# 76. A previously hypertensive 34-year-old post-partum woman who presented with right upper-quadrant pain, pelvic pain and hypotension was sent for an urgent CT. The CT scan showed a large, subcapsular liver hematoma and hemoperitoneum within the abdomen and pelvis. No liver lesions were seen on a prepartum US. What is the diagnosis?
A. Haemorrhagic adenoma
B. HELLP syndrome
C. Iatrogenic injury at caesarean section
D. Acute hepatic steatosis of pregnancy
E. Fitz-Hugh-Curtis syndrome

A
  1. B. HELLP syndrome

HELLP (haemolysis, elevated liver enzymes, low platelets) syndrome is one of the hypertensive disorders of pregnancy, occurring in 4%-12% of pre-eclamptic patients, from vascular endothelial injury, which results in intravascular deposition of fibrin with end organ damage and can occur prior to or after delivery. Disseminated intravascular coagulation (DIG) is seen in 20%-40% of patients. Other major complications include placental abruption, acute renal failure, pulmonary oedema, pleural and pericardial effusions, hepatic infarction, haematoma and rupture. Pelvic pain may occur in the setting of haemoperitoneum.
Spontaneous intrahepatic haemorrhage and rupture likely result from toxaemia-related vasculopathy with endothelial damage, leading to haemorrhage. Supportive therapy is usually offered initially, with capsular rupture necessitating surgery. Embolisation of the hepatic artery may be attempted. CT is important in initial diagnosis and for serial follow up. Findings may include subcapsular or intrahepatic haemorrhage, capsular rupture and areas of confluent necrosis secondary to infarction. Intravenous contrast-enhanced CT allows identification of active arterial extravasation, which cannot be seen on US.

52
Q

@# 77. A 25-year-old woman presents with pain and swelling over the sternoclavicular joints. Clinical examination reveals small pustules in the palms and soles along with facial acne. As part of the series of investigations, a bone scan shows increased uptake around the sternoclavicular joints and anterior chest wall resulting in a bull’s horn appearance. What is the diagnosis?
A. Osteomyelitis of sternoclavicular joint
B. Ankylosing spondylitis
C. SAPHO syndrome
D. POEMS syndrome
E. Soto syndrome

A
  1. C. SAPHO syndrome

The acronym SAPHO (synovitis, acne, palmoplantar pustulosis, hyperostosis and osteitis), which was coined in 1987, is applied to occurrences of a wide spectrum of aseptic neutrophilic dermatoses in association with aseptic osteoarticular lesions. The sternoclavicular joint (most frequently affected) is involved in 65%-90% of cases. Hyperostosis and osteosclerosis are characteristic findings at conventional radiography and CT. Other findings include joint erosion and ankylosis. In the presence of active lesions, MR imaging with T2-weighted or STIR sequence may depict bone marrow oedema, a feature that helps differentiate active lesions from chronic ones. The ‘bull’s head’ sign (i.e., increased radiotracer uptake in the sternal manubrium and both sternoclavicular joints at delayed scintigraphy) is highly specific for the diagnosis of SAPHO and may obviate biopsy.
Acute septic arthritis of the sternoclavicular joint is an uncommon infectious condition that is usually monoarticular and insidious in onset. Changes that are visible at CT include destruction of the articular surface, widening of the joint space, gas and fluid collections in the chest wall and mediastinum, and mediastinal soft-tissue stranding. MR images in addition may depict bone marrow oedema.
The sternoclavicular joint can be affected by osteoarthrosis, RA, and seronegative and crystalline arthropathy, which depict their respective specific features on imaging; however, imaging is often non-specific.

53
Q

@# 79. A suspected low-density lesion in the pancreas of a 59-year-old man seen on US was reassessed with CT, which confirmed the presence of a small central cyst in the proximal part of the body of the pancreas. All of the following are characteristic features of the main duct subtype of intraductal papillary mucinous neoplasms (IPMNs) of the pancreas, except
A. Long or short segments of marked stricture in the main pancreatic duct
B. Excess drainage of thick mucin from a patulous papillary orifice on endoscopic retrograde cholangiopancreatography
C. A papillary intraductal mass
D. Malignant potential
E. A nodular intraductal mass

A
  1. A. Long or short segments of marked stricture in the main pancreatic duct

IPMNs are a group of neoplasms in the biliary duct or pancreatic duct that cause cystic dilatation from excessive mucin production and accumulation. There are three main types of pancreatic IPMNs: main duct, branch duct and combined. A main duct IPMN commonly causes dilatation of the papilla, with bulging of the papilla into the duodenal lumen. Filling defects caused by mural nodules or mucin may be seen at MRCP or ERCP. At CT and MR imaging, filling defects caused by mural nodules enhance, while filling defects caused by mucin do not enhance.

54
Q

@# 81. Urgent MRI done on a patient who had recently undergone caesarean delivery showed a large (>5 cm) intermediate- to high-signal-intensity haematoma at the lower uterine incision site that communicates with the endometrium on sagittal fat-saturated T2W images. The uterine serosa was intact. What is the diagnosis?
A. Uterine dehiscence
B. Uterine rupture
C. Bladder flap haematoma
D. Subfascial haematoma
E. Endometritis

A
  1. A. Uterine dehiscence

Uterine dehiscence is characterised by incomplete rupture of the uterine wall, usually involving the endometrium and myometrium but with an intact overlying serosal layer. Uterine dehiscence is a very difficult imaging diagnosis. The presence of a bladder flap haematoma greater than 5 cm and larger pelvic haematomas should be considered abnormal and highly suspicious for uterine dehiscence in the proper clinical setting. MR imaging may be better than CT in checking for uterine dehiscence because of its multiplanar capability and greater soft-tissue contrast, with its ability to help identity an intact serosal layer. Uterine rupture is the most severe potential complication of caesarean delivery and is defined as separation of all layers of the uterine wall, including the serosal layer, with abnormal communication between the uterine cavity and the peritoneal cavity. The presence of gas/blood within the uterine defect extending from the endometrial cavity to the extrauterine parametrium in association with haemoperitoneum increases the likelihood of rupture in the appropriate clinical setting.

55
Q

@# 82. All of the following are true regarding imaging of the sternum and sternoclavicular joint, except
A. Primary osteomyelitis may suggest immunodeficiency.
B. Primary osteomyelitis is more common than secondary osteomyelitis.
C. Internal mammary artery graft has a higher association with post-sternotomy infection.
D. Sternal fracture suggests high energy trauma.
E. With regard to sternoclavicular joint dislocation, anterior is more common than posterior.

A
  1. B. Primary osteomyelitis is more common than secondary osteomyelitis.

Primary osteomyelitis of the sternum is uncommon, with only a small number of cases reported in the literature. It may occur in patients with a history of intravenous drug abuse, AIDS,
haemoglobinopathy or other immune deficiency states. Staphylococcus aureus infection is the most common cause of sternal osteomyelitis. Secondary osteomyelitis of the sternum is more common than the primary form, particularly after sternotomy. This is a potentially severe complication because of the risk of mediastinitis. Risk factors for secondary osteomyelitis include obesity, insulin-dependent diabetes and internal mammary artery grafts.
The presence of one or more sternal fractures implies high-energy trauma. The importance of sternal fractures lies in the high frequency of associated injuries. Spontaneous fractures of the sternum have occurred in the presence of a neoplasm (e.g., multiple myeloma, metastasis), strain from heavy lifting, labour and bone insufficiency (e.g., in osteoporosis).
Sternoclavicular dislocation is a rare occurrence; anterior (or presternal) displacement is more common, but posterior (or retrosternal) displacement is more frequently associated with life threatening complications caused by the compression of vital structures such as the trachea, great vessels and nerves. CT provides a detailed depiction of the sternoclavicular joint and helps characterise dislocation.

56
Q

@# 86. A 66-year-old woman with discomfort in the epigastric region was sent for US of the upper abdomen. GB and bile ducts were normal but an incidental mass was identified in the pancreas. A dual-phase CT was requested for further characterisation. The CT showed a large, lobulated, multicystic lesion with a central scar, enhancing septae and some stellate calcification. Which one of the following is the most likely diagnosis?
A. Solid pseudopapillary tumour
B. Intraductal papillary mucinous neoplasm
C. Mucinous cystic neoplasm
D. Serous cystadenoma
E. Gastrinoma

A
  1. D. Serous cystadenoma

In 70% of cases, Serous cystadenomas demonstrate a polycystic or microcystic pattern consisting of a collection of cysts (usually more than six) that range from a few millimetres up to 2 cm in size. Fine, external lobulations are a common feature, and enhancement of septa and the cyst wall may be seen. A fibrous central scar with or without a characteristic stellate pattern of calcification is seen in 30% of cases and, when demonstrated with CT or MR imaging, is highly specific and is considered to be virtually pathognomonic for serous cystadenoma. Pancreatic ductal dilatation is an uncommon finding in these patients. The macrocystic or oligocystic variant of these tumours is very uncommon and is seen in less than 10% of cases. Either of these variants can take the form of a single dominant macrocavity, in which case it will appear as a unilocular cyst or may contain fewer large (2 cm) cysts. The latter variant is classified as a macrocystic lesion and may be difficult to differentiate from a mucinous cystic tumour. Mucinous cystic neoplasms (mucinous cystadenomas) mainly involve the body and tail of the pancreas and do not communicate with the pancreatic duct At CT/MR, they are multilocular macrocystic lesions occasionally containing debris or hemorrhage. The complex internal architecture of the cyst, including septa and an internal wall, is best appreciated at MR imaging and endoscopic US, allowing differentiation from serous cystadenomas. Although peripheral eggshell calcification is not frequently seen at CT, it is specific for a mucinous cystic neoplasm and is highly predictive of malignancy. Pseudopapillary tumour is a solid tumour with a cystic component. Others in this group are islet cell tumour, pancreatic adenocarcinoma, and metastasis.

57
Q

@# 87. A 50-year-old man with chronic dyspnoea showed unresolving consolidation in both lungs on chest X-ray. Following a course of steroids, the consolidations improved and a provisional diagnosis of cryptogenic organising pneumonia was made. An HRCT was organised at this point, which showed multiple nodules and masses. All of the following are expected appearance for masses seen in COP on CT, except;
A. Sparing of subpleural region
B. Cylindrical bronchial dilatation in consolidation
C. Fissural and septal nodules
D. Small lung nodules
E. Peribronchial distribution

A
  1. C. Fissural and septal nodules

Frequently, the CT findings are far more extensive than expected from a review of the plain chest radiograph. The lung abnormalities show a characteristic peripheral or peribronchial distribution, and the lower lung lobes are more frequently involved. In some cases, the outermost subpleural area is spared. The lung opacities vary from ground glass to consolidation; in the latter, air bronchograms and mild cylindrical bronchial dilatation are a common finding. These opacities have a tendency to migrate, changing location and size, even without treatment. They are of variable size, ranging from a few centimetres to an entire lobe. Some patients present with nodular opacities on the chest radiograph. Lung volumes are preserved in most patients.

58
Q

@# 89. A 5-year-old girl known to the endocrinologists presented to the A&E department after a fall and had radiographs of her lower limb taken to exclude fractures. No fractures were identified, but there were several well defined lytic lesions in the metaphysis and diaphysis of the femur and tibia with an internal ground glass matrix. Clinical examinations revealed several irregular areas of skin pigmentation. What is your diagnosis?
A. McCune-Albright syndrome
B. Mazabraud syndrome
C. NF1
D. NF2
E. Cherubism

A
  1. A. McCune-Albright syndrome

Fibrous dysplasia is a relatively common benign skeletal disorder typically seen in adolescents and young adults. It can be monostotic (affecting one bone) or polyostotic (affecting multiple bones). Polyostotic fibrous dysplasia is commonly associated with cafe au lait spots, with irregular edges (‘coast of Maine’), in contrast to regular edges (‘coast of California’) observed in neurofibromatosis.
Multiple endocrine disorders are described in association with fibrous dysplasia. McCune- Albright syndrome refers to the triad of polyostotic fibrous dysplasia (usually unilateral), cafe au lait spots and precocious puberty. Fibrous dysplasia is also associated with hyperthyroidism, hyperparathyroidism, acromegaly, diabetes mellitus and Cushing syndrome.

Mazabraud syndrome refers to the association of polyostotic fibrous dysplasia with multiple soft-tissue myxomas, which are typically intramuscular.

Cherubism is a special form of fibrous dysplasia with symmetric involvement of both the maxilla and mandible. It typically affects men and tends to regress after adolescence.

59
Q

@# 91. A 50 year-old woman with palpable fullness in the epigastric region was sent for US of the upper abdomen. The GB and bile ducts were normal but a hypoechoic mass was identified in the pancreas. A dual phase CT was requested for further characterisation, which showed a multicystic lesion in the pancreatic head and reported possible serous cystadenoma. All of the following are expected features, except
A. Multiple small cysts (usually <2 cm)
B. High glycogen content in aspirated fluid
C. Communication with the pancreatic duct
D. Thin enhancing internal septa on MRI
E. Central stellate fibrotic scar in large lesions

A
  1. C. Communication with the pancreatic duct
    In 70% of cases, these benign tumours Serous cystadenomas demonstrate a polycystic or microcystic pattern consisting of a collection of cysts (usually more than six) that range from a few millimetres up to 2 cm in size. Fine, external lobulations are a common feature, and enhancement of septa and the cyst wall may be seen. A fibrous central scar with or without a characteristic stellate pattern of calcification is seen in 30% of cases and, when demonstrated at CT or MR imaging, is highly specific and is considered to be virtually pathognomonic for serous cystadenoma. Pancreatic ductal dilatation is an uncommon finding in these patients. In 20% of cases, these tumours are composed of microcysts in a honeycomb pattern and appear as well marginated, ‘spongy’ lesions with soft-tissue or mixed attenuation and a sharp interface with the vascular structures at CT. In patients with indeterminate CT findings, further characterisation with MR imaging or endoscopic US may be possible. At MR imaging, the microcysts may be seen as numerous discrete foci with bright signal intensity on T2-weighted images. Likewise, endoscopic US can help accurately depict these small microcysts as discrete small anechoic areas. Because of the benign nature of serous cystadenomas, some surgeons recommend imaging surveillance of microcystic tumours as being sufficient in asymptomatic patients.
    Unlike IPMN, serous or mucinous cystadenomas do not communicate with the pancreatic duct.
60
Q

@# 93. A child with a previous history of infantile spasms now presents with myoclonic seizures. She is noted to have a port wine stain on her face. An MRI scan of the brain is arranged to confirm the suspected diagnosis. What MRI findings would you expect in the suspected diagnosis?
A. Posterior fossa haemangioblastoma
B. Pial angioma demonstrated by leptomeningeal enhancement
C. Bilateral acoustic schwannomas
D. Multiple white matter T2 hyperintensities
E. Multiple T1W hyperintensities and cerebellar hypoplasia

A
  1. B. Pial angioma demonstrated by leptomeningeal enhancement

The clinical picture is very suggestive of Sturge-Weber syndrome. The major pathological abnormality in Sturge-Weber syndrome is a meningeal tangle of vessels, commonly referred to as an angioma, which is ordinarily confined to the pia mater. This pathological process consists of multiple capillaries and small venous channels that are matted together on the surface of
the brain. Non-contrast CT shows cortical calcification and atrophy. MRI will show diffuse leptomeningeal enhancement and ipsilateral choroid plexus enlargement. Cortical calcification can be demonstrated on T2 gradient echo.

The other options suggest different diagnoses respectively:

posterior fossa haemangioblastoma - VHL;

bilateral acoustic schwannomas - neurofibromatosis Type 2;

multiple white matter T2 hyperintensities - unidentified bright spots in neurofibromatosis Type 1;

multiple T1-weighted hyperintensities and cerebellar hypoplasia - neurocutaneous melanosis.

61
Q

@# 94. Contrast-enhanced CT of the abdomen done on a middle aged woman revealed a large heterogeneous retroperitoneal mass with distortion of regional anatomy and forward displacement of the posterior abdominal structures. All of the following CT signs will help distinguish a tumour arising within a retroperitoneal organ from a primary retroperitoneal tumour, except
A. Beak sign
B. Phantom organ sign
C. Embedded organ sign
D. Prominent feeding artery sign
E. Soft-tissue rim sign

A
  1. E. Soft-tissue rim sign

Before a tumour can be described as primarily retroperitoneal, the possibility that the tumour originates from a retroperitoneal organ must be excluded. When a mass deforms the edge of an adjacent organ into a ‘beak’ shape, it is likely that the mass arises from that organ (beak sign). When a large mass arises from a small organ, the organ sometimes becomes undetectable. This is known as the phantom organ sign. When a tumour compresses an adjacent plastic organ (e.g., gastrointestinal tract, inferior vena cava) that is not the organ of origin, the organ is deformed into a crescent shape (negative embedded organ sign). In contrast, when part of an organ appears to be embedded in the tumour (positive embedded organ sign), it is likely that the tumour originates from the involved organ. Hypervascular masses are often supplied by feeding arteries that are prominent enough to be visualised at CT or MR imaging, a finding that provides an important key to understanding the origin of the mass. Some tumours grow and extend into spaces between preexisting structures and surrounding vessels without compressing their lumina. Lymphangiomas, ganglioneuromas and lymphomas are examples of such tumours manifesting with the CT angiogram sign’ or ‘floating aorta sign’.
The soft-tissue rim sign is described with distal ureteric calculus.

62
Q

@# 98. A 65-year-old man with a cystic lesion of the pancreas detected on US was sent for a dual-phase pancreatic CT for further characterisation. The most commonly encountered cystic lesion of the pancreas is
A. Serous cystadenoma
B. Mucinous cystic neoplasm
C. Pancreatic pseudocyst
D. Main duct intraductal papillary mucinous neoplasm
E. Side branch intraductal papillary mucinous neoplasm

A
  1. E. Side-branch intraductal papillary mucinous neoplasm

IPMNs can be classified as main duct, branch duct (side-branch) or mixed IPMNs, depending on the site and extent of involvement. Main duct IPMN is a morphologically distinct entity and cannot be included in the discussion of pancreatic cysts. However, a side-branch IPMN or a mixed IPMN (in which a side-branch tumour extends to the main pancreatic duct) can have the morphologic features of a complex pancreatic cyst, making clear-cut distinction from a mucinous cystic neoplasm difficult. Identification of a septated cyst that communicates with the main pancreatic duct is highly suggestive of a side-branch or mixed IPMN. However, it is important to be aware that lack of communication with the main pancreatic duct at imaging does not exclude an IPMN. Currently, MR cholangiopancreatography is considered the modality of choice for demonstrating the morphologic features of the cyst (including septa and mural nodules), establishing the presence of communication between the cystic lesion and the pancreatic duct, and evaluating the extent of pancreatic ductal dilatation. Because these lesions are considered premalignant, surgical resection has typically been recommended. The occurrence of malignancy is significantly higher in main duct and mixed IPMNs than in side-branch IPMNs. Therefore, in cases of side-branch IPMN, the treatment decision should be based on the risk-benefit ratio.
taking into account the patient’s clinical presentation, age and surgical risk and the size and imaging morphologic features of the cyst.

63
Q

@# 99. Which of the following MRI sequences is most sensitive to assess normal myelination in children less than 1 year of age?
A. Fluid-attenuated inversion recovery (FLAIR)
B. T1W TSE
C. T2W TSE
D. Susceptibility-weighted imaging (SWI)
E. T2W GRE

A
  1. B. T1W TSE
    T1-weighted images are the most sensitive in children less than 1 year of age.

T2 weighted images are the most sensitive in children between the ages of 1 and 2 years, demonstrating a gradual shift from hyperintense to hypointense relative to grey matter.

The only area to remain hyperintense alter the age of 2 years, and often for quite some time, is the peritrigonal region.

FLAIR unsurprisingly follows the same pattern as T2 but lags behind somewhat. Both T1 - weighted and T2-weighted images should be used to assess myelination. All children should achieve the adult appearance of white matter by 36-40 months.

64
Q

@# 101. Owing to normal appearances on initial plain radiographs, MRI is increasingly requested in suspected stress fractures, particularly in professional athletes and keen sportspersons.
MRI performed in one such instance showed periosteal oedema and increased marrow signal on T2W fat-suppressed images with hardly any signal change on T1W images.
Considering the stages of the MR grading system for stress injuries, what would be the grade of this injury?
A. 0
B. 1
C. 2
D. 3
E. 4

A
  1. C. 2

The MRI sequences typically used are STIR, T1W, and T2W. The grading of such fractures is based on the MR findings and are as follows:
Grade 1 - Periosteal oedema on STIR, no marrow change
Grade 2 - Periosteal oedema on STIR plus marrow change on T2-weighted only
Grade 3 - Periosteal oedema on STIR plus marrow changes on T2-weighted and T1-weighted
Grade 4 - Visible fracture line

65
Q

@# 102. A 23-year-old mail presents with a congenital swelling underneath the left jaw, which has not changed over the past few years. Clinical examination reveals a horizontal scar overlying the same area. US shows multiloculated fluid-filled spaces at the site of the swelling in the neck. What is the diagnosis?
A. Cystic hygroma
B. Abscess
C. Thyroglossal cyst
D. Branchial cyst
E. Cystic metastasis

A
  1. A. Cystic hygroma

A cystic hygroma is the most common form of lymphangioma and constitutes about 5% of all benign tumours of infancy and childhood. The overwhelming majority (about 80%-90%) are detected by the time the patient is 2 years old.
These lesions are characteristically infiltrative in nature and do not respect fascial planes; hence recurrence post surgery is not uncommon.
On US scans, most cystic hygromas manifest as a multilocular predominantly cystic mass with septa of variable thickness. The echogenic portions of the lesion correlate with clusters of small, abnormal lymphatic channels. Fluid-fluid levels can be observed with a characteristic echogenic, haemorrhagic component layering in the dependent portion of the lesion. The most common pattern is that of a mass with low or intermediate signal intensity on T1-weighted images and hyperintensity on T2-weighted images. Infrequently, this lesion may be hyperintense on T1-weighted images, a finding associated with clotted blood or high lipid (chyle) content. In the case of haemorrhage, fluid-fluid levels may be observed.

66
Q

@# 104. A 76-year-old man came to the A&E department with abdominal pain. A plain radiograph of die abdomen done in casualty was reported as showing pneumatosis intestinalis without any complications. Which one of the following statements regarding the radiographic findings in adult pneumatosis intestinalis is true?
A. Pneumatosis cystoides intestinalis is characterised by linear gas collections.
B. Pneumatosis cystoides intestinalis occurs mainly in the rectum.
C. Portal venous gas usually is located in the central portion of the liver.
D. The submucosal gas in pneumatosis cystoides intestinalis can mimic colonic polyps on a barium enema when viewed en face.
E. The linear form in the small bowel allows differentiation between the benign and life-threatening forms.

A
  1. D. The submucosal gas in pneumatosis cystoides intestinalis can mimic colonic polyps on
    a barium enema when viewed en face.

Pneumatosis is the presence of gas bubbles within the wall of the involved segment of bowel.

The patterns of pneumatosis vary from circular to linear to bubble and curvilinear gas collections. Such patterns can differentiate benign and clinically worrisome pneumatosis intestinalis.

The circular form is usually benign and most often seen with pneumatosis cystoides intestinalis and affect the whole colon.

Pseudopneumatosis is more common in the right colon because of the presence of liquid fecal matter.

Linear or bubble- like air can be due to both benign and life- threatening causes, and its radiographic appearance alone does not allow differentiation between them. In PI due to benign causes, the bowel wall is usually normal.

Additional CT findings such as soft-tissue thickening of the bowel wall, free fluid and peri- intestinal soft-tissue stranding are more frequently associated with clinically worrisome than benign pneumatosis intestinalis.

Portal venous gas is differentiated from biliary gas by its characteristic tubular branching lucencies that extend to the periphery of the liver, whereas biliary air is more central.

67
Q

@# 106. Which of the following areas of the brain is normally the last to myelinate?
A. Anterior limb of the internal capsule
B. Posterior limb of the internal capsule
C. Cerebellum
D. Peritrigonal region
E. Brainstem

A
  1. D. Peritrigonal region

Myelin development correlates with functional milestones.
By 2 years, the infant brain should resemble the adult brain with the exception of the peritrigonal area, which is the last area to myelinate. This area can remain unmyelinated long after the age of 2 years. This can be seen on MRI as persistent T2-weighted and FLAIR hyperintensity and should not be confused with a pathological process. The process of myelination is predictable and follows a few simple rules: central to peripheral, caudal to rostral, dorsal to ventral. At birth, the cerebellum, brainstem and posterior limb of the internal capsule should normally be myelinated. The anterior limb should myelinate by 2 months.

68
Q

@# 107. A 50-year-old woman presents with painful wrists and clubbing. An X-ray demonstrates bilateral symmetrical periostitis involving the radius. The differential diagnosis includes hypertrophic pulmonary osteoarthropathy (HPOA). Which of the following is not a known cause of HPOA?
A. Mesothelioma
B. Pleural fibroma
C. Bronchogenic carcinoma
D. Bronchiectasis
E. Pleural plaque secondary to asbestos exposure

A
  1. E. Pleural plaque secondary to asbestos exposure

There are many causes of HPOA, most of which are pulmonary in origin: intrathoracic malignant tumours, including metastasis, mesothelioma, bronchogenic carcinoma and lymphoma; benign tumours, including benign pleural fibroma, thymoma and pulmonary haemangiomas; chronic pulmonary infection (e.g., bronchiectasis and lung abscesses); and cyanotic congenital heart disease. Extrathoracic causes include ulcerative colitis, Crohn’s disease, Whipple disease, liver disease (e.g., cirrhosis, liver abscesses and primary biliary cirrhosis) and gastric and pancreatic carcinomas. Although asbestosis (i.e., asbestos-induced pulmonary fibrosis) is a known cause of HPOA, pleural plaques are not. Radiological abnormalities most commonly affect the lower limb or forearm bones with cortical thickening and lamellar periosteal proliferation. Hypertrophic pulmonary osteoarthropathy is commonly present in conjunction with finger clubbing. If the underlying condition is treated, then this often quickly leads to remission of their symptoms and about a month later to resolution of the radiological findings.

69
Q

@# 108. A 23-year old man presented with a hard swelling of the left side of the jaw. Plain radiograph showed a well-defined oval lucent lesion in the left side of the mandible with the crown
of a unerupted lower third molar. What is the most likely diagnosis?
A. Odontogenic keratocyst
B. Dentigerous cyst
C. Dental cyst
D. Periapical cyst
E. Adamantinoma

A
  1. B. Dentigerous cyst

Odontogenic keratocysts are believed to arise from the dental lamina and other sources of odontogenic epithelium. At radiography, an odontogenic keratocyst usually appears as a unilocular, lucent lesion with smooth, corticated borders that is often associated with an impacted tooth. Such lesions are indistinguishable from dentigerous cysts at radiography. They are more likely to show aggressive growth than other odontogenic cysts and may have undulating borders and a multilocular appearance; these characteristics make odontogenic keratocysts indistinguishable from ameloblastomas.
The dentigerous (follicular) cyst is the most common type of non-inflammatory odontogenic cyst and the most common cause of a pericoronal area of lucency associated with an impacted tooth. At radiography, dentigerous cysts appear as well-defined, round or ovoid, corticated, lucent lesions around the crowns of unerupted teeth, usually third molars. The roots of the involved tooth are often outside the lesion and in mandibular bone.

70
Q

@# 109. A 55-year-old man with progressive productive cough and copious sputum was referred for an HRCT for disease characterisation. Which one of the following radiographic features distinguishes the bronchiectasis of allergic bronchopulmonary aspergillosis from that of cystic fibrosis?
A. High-density mucus plugging
B. Upper lobe predominance
C. Cylindrical-type predominance
D. Presence of associated hilar adenopathy
E. Presence of air trapping

A
  1. A. High-density mucus plugging

A ‘finger-in-glove’ pattern can be seen at chest radiography in ABPA that corresponds to mucoid bronchial impaction at chest CT. CT findings in ABPA include cystic or varicoid bronchiectasis, a tree-in-bud pattern of nodules, bronchial wall thickening and air trapping with a central or proximal upper-lobe predominance. High-attenuating (>70-100 HU) bronchial contents at CT represent fungal debris containing iron and manganese.
The classic diagnostic triad in patients with cystic fibrosis includes an abnormal sweat chloride test result and manifestations of pulmonary and pancreatic disease. Upper-lobe predominance is seen in many but not ail cases; a diffuse distribution is also a common sign.
Normal to increased lung volumes are typical in CF and indicate air trapping and small airways disease. CT images show extensive cystic and cylindrical bronchiectasis and bronchial wall and peribronchial interstitial thickening. Findings arc typically more extensive in patients with bronchiectasis due to cystic fibrosis than in patients with bronchiectasis due to other causes. Nodular opacities throughout the lungs correlate with areas of mucoid bronchial or bronchiolar impaction. ‘Tree-in-bud’ nodules indicate the diffuse bronchiolitis that typically occurs in cystic fibrosis. In addition, a mosaic pattern of attenuation secondary to air trapping due to obstructed bronchi and bronchioles is commonly seen.

71
Q

@# 114. Which of the following is the correct order of normal myelination?
A. Peripheral to central, caudal to rostral, ventral to dorsal
B. Central to peripheral, caudal to rostral, ventral to dorsal
C. Central to peripheral, rostral to caudal, dorsal to ventral
D. Peripheral to central, rostral to caudal, dorsal to ventral
E. Central to peripheral, caudal to rostral, dorsal to ventral

A
  1. E. Central to peripheral, caudal to rostral, dorsal to ventral

Myelination begins at 16 weeks in utero and is far from complete at term birth. It continues during the first 2 years of life until it reaches the level of myelination seen in a normal adult brain. Learning development coincides with myelination. CT and MRI can be used to assess myelination. On CT, unmyelinated white matter will appear hypodense. Unmyelinated white matter will appear hypointense on T1-weighted and hyperintense on T2-weighted. This can be confusing as many other conditions of interest in the neonate can cause T2-weighted hyperintensity. Knowing the pattern of normal myelination is therefore essential when interpreting paediatric neuroimaging.

72
Q

@# 116. Which one of the following statements concerning papillary and solid epithelial neoplasms of the pancreas is true?
A. They occur mainly in elderly men.
B. They are a high-grade malignant tumour.
C. Heterogeneous content is detected within the mass on CT.
D. They typically present as a small cystic mass in the pancreatic head.
E. Weight loss and jaundice are the usual presentation.

A
  1. C. Heterogeneous content is detected within the mass on CT.

This uncommon, typically benign tumour is found mainly in young, non-Caucasian women between the second and third decades of life. It seems to have a predilection for Asian and African American women, although rare cases have been reported in children and men.
Although most solid pseudopapillary tumours (SPTs) exhibit benign behaviour, malignant degeneration does occur. Patients with SPT of the pancreas are often clinically asymptomatic. They may present with a gradually enlarging abdominal mass or complain of vague abdominal pain or discomfort The abdomen is usually non-tender on palpation, but obstructive symptoms may occur if the tumour grows large enough to compress adjacent viscera.
There are usually no abnormalities in clinical laboratory tests (e.g., serum amylase levels) or in pancreatic cancer markers (e.g., CA19-9, carcinoembryonic antigen, fetoprotein). The diagnosis is not uncommonly made incidentally at abdominal examination, US or CT performed for other reasons.
SPT of the pancreas has distinctive pathologic features. The mass may occur anywhere in the pancreas but is most frequently found in the head or tail. At gross examination, the mass is usually large (mean maximum dimension 9.3 cm) and well encapsulated and contains varying amounts of necrosis, haemorrhage and cystic change.

73
Q

@# 118. A 29-year-old runner presents to his family doctor with progressive increase in forefoot pain, which is limiting his training. Initial X-rays in the A&E department were normal. A follow-up bone scan detects focal abnormality in one of the metatarsals. Which one of the following is the most likely location for a metatarsal stress fracture appearing as an area of focal linear sclerosis on radiographs?
A. Base of the first metatarsal
B. Mid-shaft of the second metatarsal
C. Head of the third metatarsal
D. Distal shaft of the fourth metatarsal
E. Base of the fifth metatarsal

A
  1. B. Mid-shaft of the second metatarsal
    Stress fractures of the metatarsals account for between 9% and 24.6% of stress fractures in athletes. Plain radiographic evaluation may reveal normal appearances. Two to three weeks after the injury, periosteal bone reaction may be visible on radiographs. MRI has high sensitivity in the diagnosis of these fractures, depicting oedema of the bone marrow and periosteum.
    A hypointense fracture line may also be visible on MRI. The most common site of metatarsal stress fractures are the distal shaft and neck of the second and third metatarsals, especially in long-distance runners. They are usually thinner than the first metatarsal and bear a large amount of weight during the ‘push off’ phase of running.

Stress fractures of the fifth metatarsal base occurring approximately 1.5 cm distal to the tubercle, at the junction of the metaphysis and diaphysis (Jones’ fracture), deserve special mention as high-performance athletes with this injury, because of their propensity to be delayed or non-union, and refracture, may benefit from early operative intervention.

74
Q

@# 115. A 24 year old man with chronic productive cough, fever, shortness of breath and recurrent chest infections shows predominantly lower-lobe bronchiectasis on HRCT. All the following differentials should be considered, except
A. Kartagener s syndrome
B. Sarcoidosis
C. Common variable immunodeficiency
D. α1-Antitrypsin deficiency
E. Chronic aspiration

A
  1. B. Sarcoidosis
    Bronchiectasis with Upper or Mid-Lung Predominance
    * Cystic fibrosis
    * ABPA
    * Sarcoidosis
    * TB
    Bronchiectasis with Anterior Predominance
    * Atypical mycobacterial infection
    * ARDS
    Bronchiectasis with Lower Lung Predominance
    * Pulmonary fibrosis
    * Chronic aspiration
    * Kartagener’s syndrome
    * Common variable immunodeficiency
    * -Antitrypsin deficiency
    Bronchiectasis with Central Predominance
    * Mounier-Kuhn syndrome
    * Williams-Campbell syndrome
75
Q

@# 112. A 28-year-old woman is referred for a CT KUB for loin pain. The study unexpectedly shows grossly enlarged and abnormal kidneys with areas of fat density tissue within it. Small cystic areas are noted within the scanned lung bases. Further investigations reveal small subependymal lesions. What is a common musculoskeletal manifestation of this disease?
A. Osteochondroma
B. Sclerotic bone lesions
C. Enostoses
D. Fibrous dysplasia
E. Enchondroma

A
  1. B. Sclerotic bone lesions

The patient has tuberous sclerosis, an inherited autosomal dominant multisystem disorder with multifocal systemic hamartomas. Osseous manifestations include cyst-like lesions, hyperostosis of the inner table of the calvaria, osteoblastic changes, periosteal new bone formation and scoliosis. These osseous lesions can occur anywhere in bone, commonly in the calvaria, short tubular bones of the hand or foot, spine and pelvis. The cyst-like lesions are usually irregularly circumscribed and have a sclerotic appearance peripherally